Saturday, July 16, 2011

So you think you understand the cosmological argument?

Most people who comment on the cosmological argument demonstrably do not know what they are talking about.  This includes all the prominent New Atheist writers.  It very definitely includes most of the people who hang out in Jerry Coyne’s comboxes.  It also includes most scientists.  And it even includes many theologians and philosophers, or at least those who have not devoted much study to the issue.  This may sound arrogant, but it is not.  You might think I am saying “I, Edward Feser, have special knowledge about this subject that has somehow eluded everyone else.”  But that is NOT what I am saying.  The point has nothing to do with me.  What I am saying is pretty much common knowledge among professional philosophers of religion (including atheist philosophers of religion), who – naturally, given the subject matter of their particular philosophical sub-discipline – are the people who know more about the cosmological argument than anyone else does. 

In particular, I think that the vast majority of philosophers who have studied the argument in any depth – and again, that includes atheists as well as theists, though it does not include most philosophers outside the sub-discipline of philosophy of religion – would agree with the points I am about to make, or with most of them anyway.  Of course, I do not mean that they would all agree with me that the argument is at the end of the day a convincing argument.  I just mean that they would agree that most non-specialists who comment on it do not understand it, and that the reasons why people reject it are usually superficial and based on caricatures of the argument.  Nor do I say that every single self-described philosopher of religion would agree with the points I am about to make.  Like every other academic field, philosophy of religion has its share of hacks and mediocrities.  But I am saying that the vast majority of philosophers of religion would agree, and again, that this includes the atheists among them as well as the theists.

I’m not going to present and defend any version of the cosmological argument here.  I’ve done that at length in my books Aquinas and The Last Superstition, and it needs to be done at length rather than in the context of a blog post.  The reason is that, while the basic structure of the main versions of the argument is fairly simple, the background metaphysics necessary to a proper understanding of the key terms and inferences is not.  It needs some spelling out, which is why Aquinas and The Last Superstition each devote a long chapter to general metaphysics before addressing the question of God’s existence.  The serious objections to the argument can in my view all be answered, but that too can properly be done only after the background ideas have been set out.  And that too is a task carried out in the books.

I will deal here with some of the non-serious objections, though.  In particular, what follows is intended to clear away some of the intellectual rubbish that prevents many people from giving the argument a fair hearing.  To get to the point(s), then:

1. The argument does NOT rest on the premise that “Everything has a cause.”

Lots of people – probably most people who have an opinion on the matter – think that the cosmological argument goes like this: Everything has a cause; so the universe has a cause; so God exists.  They then have no trouble at all poking holes in it.  If everything has a cause, then what caused God?  Why assume in the first place that everything has to have a cause?  Why assume the cause is God?  Etc.

Here’s the funny thing, though.  People who attack this argument never tell you where they got it from.  They never quote anyone defending it.  There’s a reason for that.  The reason is that none of the best-known proponents of the cosmological argument in the history of philosophy and theology ever gave this stupid argument.  Not Plato, not Aristotle, not al-Ghazali, not Maimonides, not Aquinas, not Duns Scotus, not Leibniz, not Samuel Clarke, not Reginald Garrigou-Lagrange, not Mortimer Adler, not William Lane Craig, not Richard Swinburne.  And not anyone else either, as far as I know.  (Your Pastor Bob doesn’t count.  I mean no one among prominent philosophers.)  And yet it is constantly presented, not only by popular writers but even by some professional philosophers, as if it were “the” “basic” version of the cosmological argument, and as if every other version were essentially just a variation on it.

Don’t take my word for it.  The atheist Robin Le Poidevin, in his book Arguing for Atheism (which my critic Jason Rosenhouse thinks is pretty hot stuff) begins his critique of the cosmological argument by attacking a variation of the silly argument given above – though he admits that “no-one has defended a cosmological argument of precisely this form”!  So what’s the point of attacking it?  Why not start instead with what some prominent defender of the cosmological argument has actually said?

Suppose some creationist began his attack on Darwinism by assuring his readers that “the basic” claim of the Darwinian account of human origins is that at some point in the distant past a monkey gave birth to a human baby.  Suppose he provided no source for this claim – which, of course, he couldn’t have, because no Darwinian has ever said such a thing – and suppose also that he admitted that no one has ever said it.  But suppose further that he claimed that “more sophisticated versions” of Darwinism were really just “modifications” of this claim.  Intellectually speaking, this would be utterly contemptible and sleazy.  It would give readers the false impression that anything Darwinians have to say about human origins, however superficially sophisticated, is really just a desperate exercise in patching up a manifestly absurd position.  Precisely for that reason, though, such a procedure would, rhetorically speaking, be very effective indeed.

Compare that to Le Poidevin’s procedure.  Though by his own admission no one has ever actually defended the feeble argument in question, Le Poidevin still calls it “the basic” version of the cosmological argument and characterizes the “more sophisticated versions” he considers later on as “modifications” of it.  Daniel Dennett does something similar in his book Breaking the Spell.  He assures us that the lame argument in question is “the simplest form” of the cosmological argument and falsely insinuates that other versions – that is to say, the ones that philosophers have actually defended, and which Dennett does not bother to discuss – are merely desperate attempts to repair the obvious problems with the “Everything has a cause” “version.”  As with our imaginary creationist, this procedure is intellectually dishonest and sleazy, but it is rhetorically very effective.  It gives the unwary reader the false impression that “the basic” claim made by Aristotle, Aquinas, Leibniz, et al. is manifestly absurd, that everything else they have to say is merely an attempt to patch up this absurd position, and (therefore) that such writers need not be bothered with further.

And that, I submit, is the reason why the stupid “Everything has a cause” argument – a complete fabrication, an urban legend, something no philosopher has ever defended – perpetually haunts the debate over the cosmological argument.  It gives atheists an easy target, and a way rhetorically to make even their most sophisticated opponents seem silly and not worth bothering with.  It‘s a slimy debating trick, nothing more – a shameless exercise in what I have elsewhere called “meta-sophistry.”  (I make no judgment about whether Le Poidevin’s or Dennett’s sleaziness was deliberate.  But that they should know better is beyond question.)

What defenders of the cosmological argument do say is that what comes into existence has a cause, or that what is contingent has a cause.  These claims are as different from “Everything has a cause” as “Whatever has color is extended” is different from “Everything is extended.”  Defenders of the cosmological argument also provide arguments for these claims about causation.  You may disagree with the claims – though if you think they are falsified by modern physics, you are sorely mistaken – but you cannot justly accuse the defender of the cosmological argument either of saying something manifestly silly or of contradicting himself when he goes on to say that God is uncaused.

This gives us what I regard as “the basic” test for determining whether an atheist is informed and intellectually honest.  If he thinks that the cosmological argument rests on the claim that “everything has a cause,” then he is simply ignorant of the basic facts.  If he persists in asserting that it rests on this claim after being informed otherwise, then he is intellectually dishonest.  And if he is an academic philosopher like Le Poidevin or Dennett who is professionally obligated to know these things and to eschew cheap debating tricks, then… well, you do the math.

2. “What caused God?” is not a serious objection to the argument.

Part of the reason this is not a serious objection is that it usually rests on the assumption that the cosmological argument is committed to the premise that “Everything has a cause,” and as I’ve just said, this is simply not the case.  But there is another and perhaps deeper reason.

The cosmological argument in its historically most influential versions is not concerned to show that there is a cause of things which just happens not to have a cause.  It is not interested in “brute facts” – if it were, then yes, positing the world as the ultimate brute fact might arguably be as defensible as taking God to be.  On the contrary, the cosmological argument – again, at least as its most prominent defenders (Aristotle, Aquinas, Leibniz, et al.) present it – is concerned with trying to show that not everything can be a “brute fact.”  What it seeks to show is that if there is to be an ultimate explanation of things, then there must be a cause of everything else which not only happens to exist, but which could not even in principle have failed to exist.  And that is why it is said to be uncaused – not because it is an arbitrary exception to a general rule, not because it merely happens to be uncaused, but rather because it is not the sort of thing that can even in principle be said to have had a cause, precisely because it could not even in principle have failed to exist in the first place.  And the argument doesn’t merely assume or stipulate that the first cause is like this; on the contrary, the whole point of the argument is to try to show that there must be something like this.

Different versions of the cosmological argument approach this task in different ways.  Aristotelian versions argue that change – the actualization of the potentials inherent in things – cannot in principle occur unless there is a cause that is “pure actuality,” and thus can actualize other things without itself having to be actualized.  Neo-Platonic versions argue that composite things cannot in principle exist unless there is a cause of things that is absolutely unified or non-composite.  Thomists not only defend the Aristotelian versions, but also argue that whatever has an essence or nature distinct from its existence – so that it must derive existence from something outside it – must ultimately be caused by something whose essence just is existence, and which qua existence or being itself need not derive its existence from another.  Leibnizian versions argue that whatever does not have the sufficient reason for its existence in itself must ultimately derive its existence from something which does have within itself a sufficient reason for its existence, and which is in that sense necessary rather than contingent.  And so forth.  (Note that I am not defending or even stating the arguments here, but merely giving single sentence summaries of the general approach several versions of the arguments take.)

So, to ask “What caused God?” really amounts to asking “What caused the thing that cannot in principle have had a cause?”, or “What actualized the potentials in that thing which is pure actuality and thus never had any potentials of any sort needing to be actualized in the first place?”, or “What imparted a sufficient reason for existence to that thing which has its sufficient reason for existence within itself and did not derive it from something else?”  And none of these questions makes any sense.  Of course, the atheist might say that he isn’t convinced that the cosmological argument succeeds in showing that there really is something that could not in principle have had a cause, or that is purely actual, or that has a sufficient reason for its existence within itself.  He might even try to argue that there is some sort of hidden incoherence in these notions.  But merely to ask “What caused God?” – as if the defender of the cosmological argument had overlooked the most obvious of objections – simply misses the whole point.  A serious critic has to grapple with the details of the arguments.  He cannot short-circuit them with a single smart-ass question.  (If some anonymous doofus in a combox can think up such an objection, then you can be certain that Aristotle, Aquinas, Leibniz, et al. already thought of it too.)

3. “Why assume that the universe had a beginning?” is not a serious objection to the argument.

The reason this is not a serious objection is that no version of the cosmological argument assumes this at all.  Of course, the kalām cosmological argument does claim that the universe had a beginning, but it doesn’t merely assume it.  Rather, the whole point of that version of the cosmological argument is to establish through detailed argument that the universe must have had a beginning.  You can try to rebut those arguments, but to pretend that one can dismiss the argument merely by raising the possibility of an infinite series of universes (say) is to miss the whole point.

The main reason this is a bad objection, though, is that most versions of the cosmological argument do not even claim that the universe had a beginning.  Aristotelian, Neo-Platonic, Thomistic, and Leibnizian cosmological arguments are all concerned to show that there must be an uncaused cause even if the universe has always existed.  Of course, Aquinas did believe that the world had a beginning, but (as all Aquinas scholars know) that is not a claim that plays any role in his versions of the cosmological argument.  When he argues there that there must be a First Cause, he doesn’t mean “first” in the order of events extending backwards into the past.  What he means is that there must be a most fundamental cause of things which keeps them in existence at every moment, whether or not the series of moments extends backwards into the past without a beginning.

In fact, Aquinas rather famously rejected what is now known as the kalām argument.  He did not think that the claim that the universe had a beginning could be established through philosophical arguments.  He thought it could be known only via divine revelation, and thus was not suitable for use in trying to establish God’s existence.  (Here, by the way, is another basic test of competence to speak on this subject.  Any critic of the Five Ways who claims that Aquinas was trying to show that the universe had a beginning and that God caused that beginning – as Richard Dawkins does in his comments on the Third Way in The God Delusion – infallibly demonstrates thereby that he simply doesn’t know what he is talking about.)

4. “No one has given any reason to think that the First Cause is all-powerful, all-knowing, all-good, etc.” is not a serious objection to the argument.

People who make this claim – like, again, Dawkins in The God Delusion – show thereby that they haven’t actually read the writers they are criticizing.  They are typically relying on what other uninformed people have said about the argument, or at most relying on excerpts ripped from context and stuck into some anthology (as Aquinas’s Five Ways so often are).  Aquinas in fact devotes hundreds of pages across various works to showing that a First Cause of things would have to be all-powerful, all-knowing, all-good, and so on and so forth.  Other Scholastic writers and modern writers like Leibniz and Samuel Clarke also devote detailed argumentation to establishing that the First Cause would have to have the various divine attributes.

Of course, an atheist might try to rebut these various arguments.  But to pretend that they don’t exist – that is to say, to pretend, as so many do, that defenders of the cosmological argument typically make an undefended leap from “There is a First Cause” to “There is a cause of the world that is all-powerful, all-knowing, etc.” – is, once again, simply to show that one doesn’t know what one is talking about.

5. “The argument doesn’t prove that Christianity is true” is not a serious objection to the argument.

No one claims that the cosmological argument by itself suffices to show that Christianity is true, that Jesus of Nazareth was God Incarnate, etc.  That’s not what it is intended to do.  It is intended to establish only what Christians, Jews, Muslims, philosophical theists, and other monotheists hold in common, viz. the view that there is a divine cause of the universe.  Establishing the truth of specifically Christian claims about this divine cause requires separate arguments, and no one has ever pretended otherwise.

It would also obviously be rather silly for an atheist to pretend that unless the argument gets you all the way to proving the truth of Christianity, specifically, then there is no point in considering it.  For if the argument works, that would suffice all by itself to refute atheism.  It would show that the real debate is not between atheism and theism, but between the various brands of theism.  

6. “Science has shown such-and-such” is not a serious objection to (most versions of) the argument.

There are versions of the cosmological argument that appeal to scientific considerations – most notably, the version of the kalām argument defended by William Lane Craig.  But even Craig’s argument also appeals to separate, purely philosophical considerations that do not stand or fall with the current state of things in cosmology or physics.  And most versions of the cosmological argument do not in any way depend on particular scientific claims.  Rather, they start with extremely general considerations that any possible scientific theorizing must itself take for granted – for example, that there is any empirical world at all, or any world of any sort at all.  

It is sometimes claimed (for example, by Anthony Kenny and J. L. Mackie) that some of Aquinas’s arguments for God’s existence depend on outdated theses in Aristotelian physics.  But Thomists have had little difficulty in showing that this is false.  In fact the arguments depend only on claims of Aristotelian metaphysics which can be disentangled from any outdated scientific assumptions and shown to be defensible whatever the scientific details turn out to be, precisely because (so the Thomist argues) they concern what any possible scientific theory has to presuppose.  (Naturally, I address this issue in Aquinas.)

Of course, many atheists are committed to scientism, and maintain that there are no rational forms of inquiry other than science.  But unless they provide an argument for this claim, they are merely begging the question against the defender of the cosmological argument, whose position is precisely that there are rational arguments that are distinct from, and indeed more fundamental than, empirical scientific arguments.  Moreover, defending scientism is no easy task – in fact the view is simply incoherent, or so I would argue (as I have in several previous posts).  Be that as it may, merely shouting “Science!” doesn’t prove anything.  

7. The argument is not a “God of the gaps” argument.

Since the point of the argument is precisely to explain (part of) what science itself must take for granted, it is not the sort of thing that could even in principle be overturned by scientific findings.  For the same reason, it is not an attempt to plug some current “gap” in scientific knowledge.  Nor is it, in its historically most influential versions anyway, a kind of “hypothesis” put forward as the “best explanation” of the “evidence.”  It is rather an attempt at strict metaphysical demonstration.  To be sure, like empirical science it begins with empirical claims, but they are empirical claims that are so extremely general that (as I have said) science itself cannot deny them without denying its own evidential and metaphysical presuppositions.  And it proceeds from these premises, not by probabilistic theorizing, but via strict deductive reasoning.  In this respect, to suggest (as Richard Dawkins does) that the cosmological argument fails to consider more “parsimonious” explanations than an uncaused cause is like saying that the Pythagorean theorem is merely a “theorem of the gaps” and that more “parsimonious” explanations of the “geometrical evidence” might be forthcoming.  It simply misunderstands the nature of the reasoning involved.

Of course, an atheist might reject the very possibility of such metaphysical demonstration.  He might claim that there cannot be a kind of argument which, like mathematics, leads to necessary truths and yet which, like science, starts from empirical premises.  But if so, he has to provide a separate argument for this assertion.  Merely to insist that there cannot be such an argument simply begs the question against the cosmological argument.

None of this entails that the cosmological argument is not open to potential criticism.  The point is that the kind of criticism one might try to raise against it is simply not the kind that one might raise in the context of empirical science.  It requires instead knowledge of metaphysics and philosophy more generally.  But that naturally brings us to the next point:

8. Hume and Kant did not have the last word on the argument.  Neither has anyone else.

It is often claimed that Hume, or maybe Kant, essentially had the last word on the subject of the cosmological argument and that nothing significant has been or could be said in its defense since their time.  I think that no philosopher who has made a special study of the argument would agree with this judgment, and again, that includes atheistic philosophers who ultimately reject the argument.  For example, I don’t think anyone who has studied the issue would deny that Elizabeth Anscombe presented a serious objection to Hume’s claim that something could conceivably come into existence without a cause.  Nor is Anscombe by any means the only philosopher to have criticized Hume on this issue.  I’m not claiming that everyone would agree that the objections leveled by Anscombe and others are at the end of the day correct (though I think they are), only that they would agree that it is wrong to pretend that Hume somehow ended all serious debate on the issue.  (Naturally, I discuss this issue in Aquinas.)

To take another example, Hume’s objection that the cosmological argument commits a fallacy of composition is, as I have noted in an earlier post, also greatly overrated.  For one thing, it assumes that the cosmological argument is concerned with explaining why the universe as a whole exists, and that is simply not true of all versions of the argument.  Thomists often emphasize that the argument of Aquinas’s On Being and Essence requires only the premise that something or other exists – a stone, a tree, a book, your left shoe, whatever.  The claim is that none of these things could exist even for an instant unless maintained in being by God.  You don’t need to start the argument with any fancy premise about the universe as a whole; all you need is a premise to the effect that a stone exists, or a shoe, or what have you.  (Again, see Aquinas for the full story.)  Even versions of the argument that do begin with a premise about the universe as a whole are (in my view and that of many others) not really damaged by Hume’s objection, for reasons I explain in the post just linked to.  In any event, I think that anyone who has studied the cosmological argument in any depth would agree that it is certainly seriously debatable whether Hume draws any blood here.  

In general, critics of the cosmological argument tend arbitrarily to hold it to a standard to which they do not hold other arguments.  In other areas of philosophy, even the most problematic views are treated as worthy of continuing debate.  The fact that there are all sorts of serious objections to materialist theories of the mind, or consequentialist views in ethics, or Rawlsian liberal views in political philosophy, does not lead anyone to suggest that these views shouldn’t be taken seriously.  But the fact that someone somewhere raised such-and-such an objection to the cosmological argument is routinely treated as if this sufficed to establish that the argument has been decisively “refuted” and needn’t be paid any further attention.

Jason Rosenhouse plays this game in his response to my recent post on Jerry Coyne.  Writes Rosenhouse:

Feser seems rather taken with [the cosmological argument], but there are many strong refutations to be found in the literature.  Off the top of my head, I found Mackie's discussion in The Miracle of Theism and Robin Le Poidevin's discussion in Arguing for Atheism to be both cogent and accessible.  

Does Rosenhouse really think that we defenders of the cosmological argument aren’t familiar with Mackie and Le Poidevin?  Presumably not.  But then, what’s his point?  That is to say, what point is he trying to make that doesn’t manifestly beg the question?  After all, what would Rosenhouse think of the following “objection”:

Rosenhouse seems rather taken with the materialist view of the mind, but there are many strong refutations to be found in the literature.  Off the top of my head, I found Foster’s The Immaterial Self and the essays in Koons’ and Bealer’s The Waning of Materialism to be both cogent and accessible.

Or, while we’re on the subject of what prominent mainstream atheist philosophers have said, what would he think of:

Rosenhouse seems rather taken with Darwinism, but there are many strong refutations to be found in the literature.  Off the top of my head, I found Fodor’s and Piatelli-Palmarini’s discussion in What Darwin Got Wrong and David Stove’s discussion in Darwinian Fairytales to be both cogent and accessible.

Rosenhouse’s answer to both “objections” would, I imagine, be: “Since when did Foster, Koons, Bealer, Fodor, Piatelli-Palmarini, and Stove get the last word on these subjects?”  And that would be a good answer.  But no less good is the following answer to Rosenhouse: Since when did Mackie and Le Poidevin have the last word on the cosmological argument?  

“But that’s different!” I imagine Rosenhouse would say.  But how is it different?  This brings us to one last point:

9. What “most philosophers” think about the argument is irrelevant.

Presumably, the difference is in Rosenhouse’s view summed up in another remark he makes in his post, viz. “There's a reason most philosophers are atheists” (he cites this survey as evidence).  By contrast, most philosophers are not dualists or critics of Darwinism (though in fact the number of prominent dualists is not negligible, but let that pass).  Now if what Rosenhouse means to imply is that philosophers who have made a special study of the cosmological argument now tend to agree that it is no longer worthy of serious consideration, then for reasons already stated, he is quite wrong about that.  But what he probably means to imply is rather that since most contemporary academic philosophers in general are atheists, we should conclude that the cosmological argument isn’t worth serious consideration.

But what does this little statistic really mean?  I’ll let Mr. Natural tell us what it means.  Because Rosenhouse’s little crack really amounts to little more than a fallacious appeal to authority-cum-majority.  What “most philosophers” think could be relevant to the subject at hand only if we could be confident that academic philosophers in general, and not just philosophers of religion, were both competent to speak on the cosmological argument and reasonably objective about it.  And in fact there is good reason to think that neither condition holds.

Consider first that, as I have documented in several previous posts (here, here, and here) prominent philosophers who are not specialists in the philosophy of religion often say things about the cosmological argument that are demonstrably incompetent.  Consider further that those who do specialize in areas of philosophy concerned with arguments like the cosmological argument do not tend to be atheists, as I noted here.  If expertise counts for anything – and New Atheist “Learn the science!” types are always insisting that it does – then surely we cannot dismiss the obvious implication that those who actually bother to study arguments like the cosmological argument in depth are more likely to regard them as serious arguments, and even as convincing arguments.

Now the New Atheist will maintain that the direction of causality goes the other way.  It isn’t that studying the cosmological argument in detail tends to lead one to take religious belief seriously, they will say.  It’s rather that people who already take religious belief seriously tend to be more likely to study the cosmological argument.  Of course, it would be nice to hear a non-question-begging reason for thinking that this is all that is going on.  And there is reason for doubting that this can be all that is going on.  After all, there are lots of other arguments and ideas supportive of religion that academic philosophers of religion do not devote much attention to – young earth creationism, spiritualism, and the like.  Evidently, the reason they devote more attention to the cosmological argument is that they sincerely believe, on the basis of their knowledge of it, that the argument is worthy of serious study in a way these other ideas are not, and not merely because they are predisposed to accept its conclusion.

The objection in question is also one that cuts both ways.  For why suppose that the atheist philosophers are more objective than the theist ones?  In particular, why should we be so confident that most philosophers (outside philosophy of religion) are atheists because they’ve seriously studied arguments like the cosmological argument and found them wanting?  Why not conclude instead that, precisely because they tend for other reasons to be atheists, they haven’t bothered to study arguments like the cosmological argument very seriously?  The cringe-making remarks some of them make about the argument certainly provides support for this suspicion.  (Again, I give examples here, here, and here.)

And there is other reason for suspicion.  After all, as philosophers with no theological ax to grind sometimes complain – see here and here for a few examples – their colleagues can too often be smugly insular and ill-informed about sub-disciplines outside their own and about the history of their own field.  And like other academics, they can be unreflective, dogmatic, and uninformed in their secularism.  Here too you don’t have to take my word for it.  Many prominent secular philosophers themselves have noted the same thing.  

Hence Thomas Nagel opines that a “fear of religion” seems often to underlie the work of his fellow secularist intellectuals, and that it has had “large and often pernicious consequences for modern intellectual life.”  He continues:

I speak from experience, being strongly subject to this fear myself: I want atheism to be true and am made uneasy by the fact that some of the most intelligent and well-informed people I know are religious believers.  It isn't just that I don't believe in God and, naturally, hope that I'm right in my belief.  It's that I hope there is no God!  I don't want there to be a God; I don't want the universe to be like that.  My guess is that this cosmic authority problem is not a rare condition and that it is responsible for much of the scientism and reductionism of our time.  One of the tendencies it supports is the ludicrous overuse of evolutionary biology to explain everything about human life, including everything about the human mindThis is a somewhat ridiculous situation… [I]t is just as irrational to be influenced in one’s beliefs by the hope that God does not exist as by the hope that God does exist. (The Last Word, pp. 130-131)

Jeremy Waldron tells us that:

Secular theorists often assume they know what a religious argument is like: they present it as a crude prescription from God, backed up with threat of hellfire, derived from general or particular revelation, and they contrast it with the elegant complexity of a philosophical argument by Rawls (say) or Dworkin.  With this image in mind, they think it obvious that religious argument should be excluded from public life... But those who have bothered to make themselves familiar with existing religious-based arguments in modern political theory know that this is mostly a travesty... (God, Locke, and Equality, p. 20)

Tyler Burge opines that “materialism is not established, or even clearly supported, by science” and that its hold over his peers is analogous to that of a “political or religious ideology” (“Mind-Body Causation and Explanatory Practice,” in John Heil and Alfred Mele, eds., Mental Causation, p. 117)

John Searle tells us that “materialism is the religion of our time,” that “like more traditional religions, it is accepted without question and… provides the framework within which other questions can be posed, addressed, and answered,” and that “materialists are convinced, with a quasi-religious faith, that their view must be right” (Mind: A Brief Introduction, p. 48)

William Lycan admits, in what he himself calls “an uncharacteristic exercise in intellectual honesty,” that the arguments for materialism are no better than the arguments against it, that his “own faith in materialism is based on science-worship,” and that “we also always hold our opponents to higher standards of argumentation than we obey ourselves.” (“Giving Dualism its Due,” a paper presented at the 2007 Australasian Association of Philosophy conference at the University of New England)

The atheist philosopher of religion Quentin Smith maintains that “the great majority of naturalist philosophers have an unjustified belief that naturalism is true and an unjustified belief that theism (or supernaturalism) is false.”  For their naturalism typically rests on nothing more than an ill-informed “hand waving dismissal of theism” which ignores “the erudite brilliance of theistic philosophizing today.”  Smith continues:

If each naturalist who does not specialize in the philosophy of religion (i.e., over ninety-nine percent of naturalists) were locked in a room with theists who do specialize in the philosophy of religion, and if the ensuing debates were refereed by a naturalist who had a specialization in the philosophy of religion, the naturalist referee could at most hope the outcome would be that “no definite conclusion can be drawn regarding the rationality of faith,” although I expect the most probable outcome is that the naturalist, wanting to be a fair and objective referee, would have to conclude that the theists definitely had the upper hand in every single argument or debate.

Due to the typical attitude of the contemporary naturalist… the vast majority of naturalist philosophers have come to hold (since the late 1960s) an unjustified belief in naturalism. Their justifications have been defeated by arguments developed by theistic philosophers, and now naturalist philosophers, for the most part, live in darkness about the justification for naturalism. They may have a true belief in naturalism, but they have no knowledge that naturalism is true since they do not have an undefeated justification for their belief.  If naturalism is true, then their belief in naturalism is accidentally true.  [“The Metaphilosophy of Naturalism,” Philo: A Journal of Philosophy (Fall-Winter 2001)]

Again, Nagel, Waldron, Burge, Searle, Lycan, and Smith are not apologists for religion.  Apart from Smith, they aren’t even philosophers of religion.  All of them are prominent, and all of them are “mainstream.”  They have no motive for saying the things they do other than that that is the way things honestly strike them based on their knowledge of the field.

But scientists shouldn’t get smug over lapses in objectivity among philosophers.  For at least where philosophical matters are concerned, many scientists are hardly more competent or objective, as we have seen in an earlier post, and as the embarrassing philosophical efforts of Richard Dawkins and Stephen Hawking illustrate.  And if you think even their “purely scientific” pronouncements are always free of anything but good old tough-minded “just the facts, ma’am” objectivity… well, as Dawkins will tell you, you shouldn’t believe fairy tales.  Biologist Richard Lewontin let the cat out of the bag some time ago:

Our willingness to accept scientific claims that are against common sense is the key to an understanding of the real struggle between science and the supernatural.  We take the side of science in spite of the patent absurdity of some of its constructs, in spite of its failure to fulfill many of its extravagant promises of health and life, in spite of the tolerance of the scientific community for unsubstantiated just-so stories, because we have a prior commitment, a commitment to materialism.  It is not that the methods and institutions of science somehow compel us to accept a material explanation of the phenomenal world but, on the contrary, that we are forced by our a priori adherence to material causes to create an apparatus of investigation and a set of concepts that produce material explanations, no matter how counterintuitive, no matter how mystifying to the uninitiated.  Moreover, that materialism is absolute, for we cannot allow a Divine Foot in the door.  [From a review of Carl Sagan's The Demon-Haunted World in the New York Review of Books (January 9, 1997)]

But here’s the bottom line.  The “What do respectable people say?” stuff that Rosenhouse, Coyne, and other New Atheists are always engaging in is juvenile, and futile too, since they are never able to tell us what counts as “respectable” in a way that doesn’t beg all the questions at issue.  It is amazing how much time and energy New Atheist types put into trying to come up with ever more elaborate excuses for not engaging their critics’ actual arguments.  If that alone doesn’t make you suspicious, then I submit that you are not thinking critically.

Addendum: For two followup posts in reply to Jason Rosenhouse, go here and here.

512 comments:

  1. dguller,

    I think you are right. But it misses the point of what I was trying to say. Did you read the article of Feser's that I linked to?

    Take note that I said: "claiming that reason is limited with regard to any cosmological argument in principle is not an option." The key words here are "in principle".

    I am mainly interested in commenting on your view about the scope of reason and brute facts being possible in the case of the existence of God.

    I am talking about logical implications not psychological claims.

    ReplyDelete
  2. GentleSkeptic,

    I count fifty-some-odd paragraphs explaining the ridiculous ways in which various less-gifted folks have misunderstood a simple argument.

    Perhaps Feser could just present the argument?


    Reading comprehension not your strong suit? From Feser's post:

    "I’m not going to present and defend any version of the cosmological argument here. I’ve done that at length in my books Aquinas and The Last Superstition, and it needs to be done at length rather than in the context of a blog post. "

    ReplyDelete
  3. Perhaps Feser could just present the argument?

    Please lurk more, follow the links.

    The smarter you are, the harder you have to work to believe.

    Cute. I've known lots of not-very-intelligent people who never bothered to put a speck of effort in their disbelief...

    ReplyDelete
  4. @Gentle Skeptic
    Feser has presented his argument. In this blog Feser says, "I’m not going to present and defend any version of the cosmological argument here. I’ve done that at length in my books Aquinas and The Last Superstition, and it needs to be done at length rather than in the context of a blog post."

    ReplyDelete
  5. Feser's version of the cosmological argument is provably wrong:

    "What defenders of the cosmological argument do say is that what comes into existence has a cause … You may disagree with the claims–though if you think they are falsified by modern physics, you are sorely mistaken"

    Anyone with passing familiarity with physics can see this for themselves. For example, here is Feynman in his popular QED on the causality of creation in physics (p. 58f, emph. mine):

    "quantum electrodynamics, which looks at first like an absurd idea with no causality, no mechanism, and nothing real to it, produces effects that you are familiar with: light bouncing off a mirror, light bending when it goes from air into water, and light focused by a lens. It also produces other effects that you may or may not have seen, such as the diffraction grating and a number of other things. In fact, the theory continues to be successful at explaining every phenomenon of light."

    Feser should acquaint himself with with these basic physical facts before making grandiose, incorrect statements about causality. As an exercise, he can start with explaining to us the causality of electron-electron repulsion, or beta-decay, both represented by these Feynman diagrams:

    https://secure.wikimedia.org/wikipedia/en/wiki/File:MollerScattering-t.svg
    https://secure.wikimedia.org/wikipedia/en/wiki/File:Beta_Negative_Decay.svg

    As a defender and author of two books on the cosmological argument, perhaps Feser can tell us in a paragraph or less what causes the creation of a photon? What cause the creation of a positron-electron pair? What causes the creation of a W boson? Causality is meaningless in the context of the physical description of these objects, and this is the context that describes our everyday reality to almost one part in a billion billion. Feser includes Feynman's photo and anecdotes in a post about physics, but ignores Feynman's physics, which make mincemeat of Feser's notion of causality and the cosmological argument.

    If Feser does step up and explain what creates a photon and the rest, be sure to compare his answer to Feynman's, again from QED, p. 92f:

    "let's calculate the probability that two electrons, at points 1 and 2 in space-time, end up at points 3 and 4 (see Fig. 59). … To make a more exact calculation that will agree more closely with the results of experiment, we must consider other ways this event could happen. For instance, for each of the two main ways the event can happen, one elec­tron could go charging off to some new and wonderful place and emit a photon (see Fig. 60). Meanwhile, the other electron could go to some other place and absorb the photon. Calculating the amplitude for the first of these new ways involves multiplying the amplitudes for: an electron goes from 1 to the new and wonderful place, S (where it emits a photon), and then goes from 5 to 3; the other electron goes from 2 to the other place, 6 (where it absorbs the photon), and then goes from 6 to 4. … But wait: positions 5 and 6 could be anywhere in space and time—yes, anywhere—and the arrows for all of those positions have to be calculated and added together. …
    I would like to point out something about photons being emitted and absorbed: if point 6 is later than point 5, we might say that the photon was emitted at 5 and absorbed at 6 (see Fig. 61). If 6 is earlier than 5, we might prefer to say the photon was emitted at 6 and absorbed at 5, but we could just as well say that the photon is going backwards in time! However, we don't have to worry about which way in space-time the photon went; it's all included in the for­mula for P(5 to 6), and we say a photon was "exchanged." Isn't it beautiful how simple Nature is!"

    ReplyDelete
  6. Sgt Pauper said:

    "If the universe is a set of all things, then it stands that it cannot be a thing per se because that would make the universe a member of its own set.
    Now normally, you could call a set that occurs within the universe a thing (or from outside the set concerned). But the universe is a collection of everything; there is no 'outside' from which we may call it a thing"

    Only a set which contains all sets that don’t contain themselves create logical contradiction, or the Russell's paradox - as far as I remember.
    Otherwise a set can contain itself and there are infinity of such sets. Or, as the case is often in the theory of sets, opinions vary.

    .. But the universe is a collection of everything; there is no 'outside' from which we may call it a thing…

    If there is no “outside” from which we may call it a thing then any general statement about universe can not be accepted for exactly the same reason.

    Anonymous listener

    ReplyDelete
  7. Steve:

    Just a quick question from someone with an amateur familiarity with quantum theory and particle physics: At the level of decay that you describe, is there anything happening prior to and nearby the particles that ultimately decay? Is there energy around them, forces around them, other particles around them, real or virtual?

    I could see your point if there was absolutely nothing existing prior to and surrounding the particle that decays except for the particle that decays, but it seems that there is a lot going on around the decaying particle, and that all those forces, energies and particles might result in the very instability that makes the particle decay in the first place.

    An uncaused event would have to be an ex nihilo event, i.e. caused by absolutely nothing at all. It seems that there are a number of causal influences acting upon those particles that decay, and that although none are determinate, it does not necessarily follow that none are doing anything at all to influence the decay.

    It might be the case that we just do not know how the antecedent conditions combine to result in the decay, but it does not follow that they are not doing anything at all.

    Any thoughts?

    ReplyDelete
  8. Michael:

    >> I think you are right.

    Then let’s just leave it at that. ;)

    >> But it misses the point of what I was trying to say.

    Crap.

    >> Did you read the article of Feser's that I linked to?

    I read it a while ago.

    >> Take note that I said: "claiming that reason is limited with regard to any cosmological argument in principle is not an option." The key words here are "in principle".

    Why not? I suppose that it would be helpful to explain my understanding of logic and reason.

    Logic and reason are rules of inference that we have abstracted from the patterns and regularities in the natural world. When applied to the natural world, these tools are spectacularly successful and absolutely necessary to know anything at all about that world. As such, they are now indubitable and essential components of our conceptual framework from which we can interpret and understand anything at all.

    My skepticism is whether this marvelous tool is necessarily reliable at all levels of reality, including the deepest and most fundamental one. That would presuppose that the regularities and patterns that apply in the empirical world equally occur at the deepest level of reality.

    In all honesty, I do not even know how one would go about demonstrating this, because the very demonstration would require the rules of logic and reason, which we are trying to see through. However, because we cannot make this demonstration does not mean that we necessarily get to conclude that logic applies to all levels of reality. It may be that it is just how we understand the world, and not be representative in all contexts.

    It also does not mean that logic and reason are arbitrary and do not apply to any level of reality. Clearly, they operate extremely well to our empirical experience of the world, and thus are essential in that domain.

    I’m not too sure if this helps, but I think you should know where my intuitions lie. I just worry that we not make the mistake of thinking that the world is blue simply because we only have blue photoreceptors in our retinas, and this occurred, because we evolved in an environment that only has blue objects. However, what if we now move into a new environment with green objects? We will still only see blue, because green is just a combination of blue and yellow wavelengths, and will mistakenly think that everything is blue when the reality is otherwise.

    ReplyDelete
  9. David Stoecker,

    Welcome to the forums. Thank you for sharing your story; I hope you continue to stick around these parts and pick up on some of the philosophy going on around here.

    With that said, notice that "apologetics" as in the sense of a defense of being Christian is not what this forum is about. It's about philosophy and argumentation and where the truth leads.

    You will notice that there are some people who find certain versions of the cosmological argument convincing and still are not Christian.

    The cosmological argument doesn't logically entail Christianity.

    ReplyDelete
  10. is there anything happening prior to and nearby the particles that ultimately decay?

    No. Feynman answers all these questions, so I’ll just parrot him from QED, pp. 18f:

    "Another possible theory is that the photons have some kind of internal mechanism-”wheels” and “gears” inside that are turning in some way-so that when a photon is “aimed” just right, it goes through the glass, and when it’s not aimed right, it reflects. … The trouble with that theory is, it doesn’t agree with experiment … Try as we might to invent a reasonable theory that can explain how a photon “makes up its mind” whether to go through glass or bounce back, it is impossible to predict which way a given photon will go. Philosophers have said that if the same circumstances don’t always produce the same results, predictions are impossible and science will collapse. Here is a circumstance—identical photons are always coming down in the same direction to the same piece of glass—that produces different results. We cannot predict whether a given photon will arrive at A or B. All we can predict is that out of 100 photons that come down, an average of 4 will be reflected by the front surface. Does this mean that physics, a science of great exactitude, has been reduced to calculating only the probability of an event, and not predicting exactly what will happen? Yes. That’s a retreat, but that’s the way it is: Nature permits us to cal­culate only probabilities. Yet science has not collapsed."

    There is no “cause” for the creation of a photon (or W boson or electron or quark or any physical particle), and causality doesn’t even make sense in the context of quantum field theory, in which particles can traverse any continuous path in space-time, forward or backward in time, in or out of the light cone.

    ReplyDelete
  11. "what is contingent has a cause."

    How is this not a tautology?

    ReplyDelete
  12. Steve:

    >> No.

    So, there are no surrounding particles, no energy or forces operative? It seems that nothing exists except a single subatomic particle. Is that what Feynman is saying?

    >> There is no “cause” for the creation of a photon (or W boson or electron or quark or any physical particle),

    But is that because the causal factors are too numerous, each of which is exerting a miniscule effect? There is no cause for a dice landing on 6, but that is not because there are no antecedent causes, but because there are too many, e.g. air resistance, rotational energy of the dice, force of gravity, and so on.

    ReplyDelete
  13. dguller,

    Thank you for responding to my post and clarifying your position.

    So the statement “it is possible that the laws of logic do not apply at all levels of reality” could at some level of reality be false? Well in that case...

    I understand your concern about the rose colored glasses. Is it from Kant? Any other modern philosophers? It is my position that such a concern only comes when you deny the general reliability of our senses and are pulled into an ideology (through use of reason, no less). But it is my experience (so far at least) that any such trap can be refuted so that one is never in such a position.

    Also, it may be just me, but I get the impression that you sometimes fluctuate on the empirical foundations of the cosmological argument. It's foundations are empirical (at least Aquinas' version), rooted in observation of the world (however general they may be). Am I just mistaken and you already know this?

    ReplyDelete
  14. @dguller,

    Logic and reason are rules of inference that we have abstracted from the patterns and regularities in the natural world.

    That is a naturalist's assumption. The question you are asking (whether reason applies in supernatural context) requires one to also question this assumption.

    On a whimsical (perhaps) note, on the question whether quantum indeterminacy might subvert the cosmological argument, consider:

    Either (1) the subatomic particle is affected by external actual Y to (whatever), or (2) it isn't. If (1) then obviously A-T hylemorphism covers it. If (2), then it qualifies as an animate body, and so A-T hylemorphism covers it.

    "What is to us uncertainty is, to the electron, freedom" (Arthur M. Young).

    ReplyDelete
  15. there are no surrounding particles, no energy or forces operative? … Is that what Feynman is saying?

    No. Yes (QED, pp. 85f):

    "I present to you the three basic actions, from which all the phenomena of light and electrons arise.

    –ACTION #1: A photon goes from place to place.

    –ACTION #2: An electron goes from place to place.

    –ACTION #3: An electron emits or absorbs a photon.

    Each of these actions has an amplitude—an arrow—that can be calculated according to certain rules. In a moment, I'll tell you those rules, or laws, out of which we can make the whole world"

    All you need to know to describe the entirety of the reality in front of you is how to compute the probabilities of electron-photon couplings. And there is no such thing as "force"—this a classical term for what we now "know" (tested to one part in a billion billion) to be particle-particle interactions. That's what Feynman is saying.

    A used copy will set you back $1.76.

    ReplyDelete
  16. Dear GentleSceptic:
    "Perhaps Feser could just present the argument?"

    Dr. Feser HAS presented the argument, namely in 'Aquinas' and 'The Last Superstition'.

    ReplyDelete
  17. Oh please, not the whole "does caused=determined?"-story again. Didn't everyone and their mother already read about that when the compatibilists waged war on the libertarians?

    ReplyDelete
  18. "Only a set which contains all sets that don’t contain themselves create logical contradiction"

    Yes, but that contradiction can only be removed by doing away with the assumption that such a set can be constructed. The problem with naïve set theory is exactly the assumption that there is a "set of all sets".

    ReplyDelete
  19. dguller: "At this time, no-one is aware of any causes of radioactive decay. They do not occur in QM models."

    Although this objection has been answered more than adequately on a technical level by Al Moritz... my simplistic brain says: "Doesn't radioactive decay require radioactive material?"

    How could it be "uncaused" (even theoretically) if it cannot occur unless something else exists?

    ReplyDelete
  20. GentleSkeptic: Perhaps Feser could just present the argument?

    Again?? What's wrong with all the other posts where he discusses it, or the book he wrote which explains it in detail, or all the other books that offer competent explanations, or the other websites, including Aquinas's actual presentation of the argument? If those aren't enough for you, another post here isn't going to help, and if those are sufficient, what are you asking for?



    The smarter you are, the harder you have to work to believe.

    ... that you're the centre of the universe? that stuff just happens for no reason at all? that science is magic? I dunno, you'd think so, but there are at least a few atheists who are pretty intelligent and yet somehow believe things like that anyway.

    ReplyDelete
  21. @GentleSkeptic
    He's done it countless times at this blog. Just look around. You could also purchase either his "The Last Superstition" or "Aquinas" for a fuller treatment. Neither are expensive and both are worth more than the price.

    ReplyDelete
  22. Steve Smith: Anyone with passing familiarity with physics can see this for themselves.

    Maybe more than a passing familiarity would be helpful. Or are you trying to claim that Feynman thought QM was "an absurd idea with no causality and nothing real to it"? Actually, the real problem is that one needs at least a passing familiarity with the relevant philosophical meaning of "cause". 



    As an exercise, he can start with explaining to us the causality of electron-electron repulsion

    An electron has a "charge", which causes it to attract or repel other charged particles (exchange of photons, see QED p. 91). If an electron didn't have a charge or had a different charge, then something different would happen. That's what causality is, and electrons got it. More generally, "causation" is the kind of thing that happens when you can do experiments and calculations; in other words, the very success of QM proves the kind of causation Feser is speaking of.


    You should acquaint yourself with with these basic metaphysical facts before making grandiose, incorrect statements about causality.

    ReplyDelete
  23. are you trying to claim that Feynman thought QM was "an absurd idea with no causality and nothing real to it"?

    Yes, or rather I quoted Feynman directly. QED has no causality, appears to be absurd and unreal based on our macroscopic intuitions, and yet describes our entire world very, very accurately. Anyone with a passing familiarity of the subject knows these basic features that Feyman describes.

    ReplyDelete
  24. Steve:

    There are two main problems that I have with your line of reasoning.

    First, take the example of flipping a coin. One can calculate the probability of the coin landing on heads or on tails. This probability calculation is incredibly accurate. According to your line of reasoning, it follows that this is an uncaused event. This is clearly untrue, because the coin is affected by a number of causes prior to landing on either heads or tails. So, the fact that a probability calculation can be incredibly accurate in predicting the probability of outcomes does not imply that the ultimate event is uncaused in the requisite sense of being an ex nihilo event.

    Second, take the example of Newtonian mechanics. This model of motion of macroscopic bodies is incredibly accurate in its respective domain. It does not make any mention of the atoms that compose the physical bodies that are involved. According to your line of reasoning, because it is a highly successful scientific model, if it is silent about atoms, then it follows that atoms do not exist and are not involved. This clearly does not follow, and if it does not work in this case, then it cannot work in your case. After all, just because QM makes no mention of specific antecedent causal conditions does not necessarily mean that there are no antecedent conditions.

    From what I understand, QM seems to prohibit ex nihilo events altogether. Even in a vacuum, there is energy, and thus there is always something causing something else to happen. There is never absolutely nothing, and thus there is no such thing as an uncaused ex nihilo event, even in QM. However, perhaps someone who knows QM in depth would be able to speak to this issue better than I can.

    ReplyDelete
  25. the very success of QM proves the kind of causation Feser is speaking of

    No. Feser says "what comes into existence has a cause". The photons that "come into existence" in QED have no cause, and Feser's statement is nonsensical in this framework. Answer the questin: what caused the photon?As Feynman points out in the quoted passages, this question has no meaningful answer. But please don't let a little thing like that stop you or Feser from applying discredited Thomist notions of causality to quantum field theory.

    Also, you have conflated electrical charge with the junction value J of electron-photon coupling. Charge is a consequence of this coupling. Finally, this isn't metaphysics—it's simply physics.

    ReplyDelete
  26. Steve:

    And I think that Feynman is confusing unpredictable with uncaused. Just because we cannot predict why one event amongst several possible events happens, according to any current scientific model, does not mean that the event is uncaused. I think that it is more likely that there are underlying causes that we just haven’t figured out yet, and perhaps never will. I don’t think that there is any valid form of reasoning that would demonstrate this conclusion.

    ReplyDelete
  27. I think that it is more likely that there are underlying causes that we just haven’t figured out yet

    No. This is ruled out by Bell's inequality, and again, Feyman addresses this possibility in one of the passages I quoted. You may as well conjecture that the particles that regularly pop into existence are caused by the angels to which Aquinas devotes much of his theological work.

    Rather than making stuff up to fit a preconceived and incorrect description of the world, try reading a little background that accurately describes the world as we actually know it to be.

    ReplyDelete
  28. Steve:

    Didn't Bohm show that one could have an interpretation of QM that contained hidden variables?

    ReplyDelete
  29. Steve:

    And all Feynman seems to be saying is that thus far, we have been unable to discover the hidden causes behind the quantum phenomena in question, and thus they are not present in his model. Does it follow that if X is not in a scientific model, then X does not exist? Care to provide an argument for that proposition?

    ReplyDelete
  30. Daniel Smith: "Doesn't radioactive decay require radioactive material?" How could it be "uncaused" (even theoretically) if it cannot occur unless something else exists?

    Indeed. If it's physical, it has a material cause. And if you can calculate it with formulas, then it has a formal cause. And if you can predict it — even statistically! — then it has a final cause. The problem is people who know no philosophy think the only cause there is is efficient causation... actually, worse, they think it's billiard-ball causation. Then, when QM disproves billiard-ball causality (taking only a few millennia to catch up with Aristotle), they run around shrieking, "Aieeee! No causality, no causality!!!"

    ReplyDelete
  31. Steve Smith: Yes, or rather I quoted Feynman directly. QED has no causality, appears to be absurd and unreal based on our macroscopic intuitions

    You should read your own quotation, maybe even the whole context. It LOOKS AT FIRST LIKE causeless absurdity. How "unreal" it may seem rather depends on one's previous philosophical expectations, but the point is that QM works. Feynman did not devote his career to doing imaginary physics, and if you think QM is not real, I can assure you it is real physics and it works very well.

    Feser says "what comes into existence has a cause". The photons that "come into existence" in QED have no cause, and Feser's statement is nonsensical in this framework.

    If by "this framework" you mean, "using different definitions to pretend he said something completely different from what he meant", then sure. Why don't you deal with the actual argument being presented, though? That you do not understand Aquinas's notions of causality hardly discredits them. Why don't you explain the causes, and show how particle physics escapes each one? It should be easy, if you claim has any basis.



    Finally, this isn't metaphysics—it's simply physics.

    Aha, that explains where you went wrong — Feser is a philosopher, and this is a site about philosophy, so it is metaphyiscs. You will find that the argument makes a lot more sense when you apply the correct terminology.

    ReplyDelete
  32. Didn't Bohm show that one could have an interpretation of QM that contained hidden variables?

    Yes, but the experiments are not in favor of causal nonlocal theories:

    http://en.wikipedia.org/wiki/Bell_test_experiments#Gr.C3.B6blacher_et_al._.282007.29_test_of_Leggett-type_non-local_realist_theories

    ReplyDelete
  33. Why don't you explain the causes, and show how particle physics escapes each one?

    To repeat, I'm the one adopting Feynman's treatment of QED as noncausal, so it's missing the point to ask me to explain causation in QED.

    But Feser should be able to. I'll repeat the question. What caused the photon's existence in these two Feynman diagrams? What caused the W boson's existence?

    https://secure.wikimedia.org/wikipedia/en/wiki/File:MollerScattering-t.svg
    https://secure.wikimedia.org/wikipedia/en/wiki/File:Beta_Negative_Decay.svg

    Feser is a philosopher, and this is a site about philosophy, so it is metaphyiscs.

    Metaphysics that violates physics is provably wrong. Feser's description of causality violates the principles quantum field theory, so Feser is wrong.

    ReplyDelete
  34. Steven:

    Right, but if you eliminate locality, then you can still have hidden variables, which means that there may be underlying causal factors explaining the quantum phenomena. At least, that's my understanding of it.

    ReplyDelete
  35. @Steve Smith,

    You said, "Metaphysics that violates physics is provably wrong. Feser's description of causality violates the principles of quantum field theory, so Feser is wrong."

    I'm confused by your phrase, "Feser's description of causality." You seem to be referring to Feser saying, "what comes into existence has a cause." When you say that according to Feynman, QED is noncausal, are you approaching the question from the view of efficient causation only? If so, this would illustrate Dr. Feser's point in this post ("The interaction problem," Oct. 8, 2008): "For the moderns, all causation gets reduced to what the Aristotelians called efficient causation; that is to say, for A to have a causal influence on B is for A either to bring B into being or at least in some way to bring into existence some modification of B. Final causality is ruled out; hence there is no place in modern thought for the idea that B might play an explanatory role relative to A insofar as generating B is the end or goal toward which A is directed. Formal causality is also ruled out; there is no question for the moderns of a material object’s being (partially) explained by reference to the substantial form it instantiates. We are supposed instead to make reference only to patterns of efficient causal relations holding between basic material elements (atoms, or corpuscles, or quarks, or whatever)." While I'm sure you already know this, for the benefit of others and to avoid misunderstanding, I would add that the phrase "the end or goal toward which A is directed" in no way asserts that natural entities decide or desire to act. No conscious thought or intention is implied in Aristotelian goal directedness. (See TLS, pgs. 69-70)

    Because I'm not a scientist or a philospher, and you Steve seem very much to be at least the former, I was disappointed that you wouldn't answer Mr. Green's challenge. ("Why don't you explain the causes, and show how particle physics escapes each one?") I for one could benefit from more knowledge about the nature or essence of the photon in order to understand your point (and Feynman's) about QED as being noncausal. Perhaps if you could explain the photon in terms of potentiality and actuality, or in terms of its essence in addition to its existence, I could better grasp your argument. I take it that the photon is coming into and out of existence seemingly randomly from our perspective. However, having as part of its essence the quality of randomly coming into and out of existence is not the same as Pure Actuality, which is ultimately what we are attempting to describe as the First Cause or Uncaused Cause.

    Earlier posts discussed apples, humans and radioactive elements. These all have natures or essences such that they are constantly coming into existence and going out of existence, on different time frames. These things are composites of essence and existence. Is that also true of a photon?

    ReplyDelete
  36. @dguller

    To add to your comment on hidden variables, I found this post of Dr. Feser's to be helpful, "The early Wittgenstein on scientism," June 1, 2010: "The supposition that “the so-called laws of nature are the explanations of natural phenomena” is an “illusion” for two reasons (which do not necessarily correspond to Wittgenstein’s reasons). First, “laws of nature” are mere abstractions and cannot explain anything. What exist in the natural order are concrete material substances with certain essences, and talk of “laws of nature” is merely shorthand for the patterns of behavior they tend to exhibit given those essences. Second, that some fundamental level of material substances (basic particles, or whatever) exist and behave in accordance with such laws can also never be the ultimate explanation of anything, because we need to know, not only how such substances came into existence, but what keeps them in existence. For as compounds of act and potency and essence and existence, they cannot possibly account for themselves; only that which is Pure Act and Subsistent Existence Itself can be the ultimate explanation of them, or of anything else. In general, whatever is composite in any way requires explanation in terms of that which is metaphysically simple. (As usual, see The Last Superstition and Aquinas for the full story.)"

    I think it will make more sense if Steve Smith will be so kind as to answer my previous post and explain the essence of photons since I'm very much in the dark on that question. (I'm new to this blog. Are bad puns excused after midnight?)

    ReplyDelete
  37. Re: Quantum physics.

    At best, all empirical investigation could get us to is a situation where we observe things at given times that weren't there previously. We never see anything 'come into existence totally uncaused from nothing'. In the case of quantum physics, there's plenty of reason to suspect that particular things we see either lack a physical cause, or our understanding of the physical is gravely mistaken.

    Neither will trouble anyone who isn't a materialist. And that some materialists are willing to bite the bullet and say "it couldn't have had a physical cause, and any possible cause would require an altered metaphysics or an immaterial cause, which I don't accept" is of no consequence to someone who rejects materialism or accepts immaterial causes.

    Though I love the move to not only insist science is not provisional, but that an unproven and unproveable interpretation of empirical data is absolute truth. The people who celebrate science the loudest seem happy to abuse it so often.

    ReplyDelete
  38. Martin,

    "An electron does not have the potential to become a human being."

    I tend to agree. But I'm following the implications of this actuality/potentiality thing. Feser asserts that form must mix with matter or all would dissolve away. So if form mixes with matter, what part of matter does it mix with? If not electrons, what? It seems to me that if we take Feser seriously we do indeed have to have "human" electrons. We also have to have "apple" electrons. And since electrons appear to be the same, we have electrons that are both "human" and "apple" in both humans and apples. It's bizarre, I admit it.

    ReplyDelete
  39. Can you explain to me why atheists like to "refute" arguments before knowing the full case for them?

    No, no Martin, you got that wrong. You should have said:

    'Can you explain to me why atheists like to "refute" arguments that nobody is actually making?'

    DaveElectric:

    God is something a lot more specific than an uncaused cause. God is a supernatural consciousness that transcends the laws of the universe and often is assumed to possess certain attributes such as omniscience, omnipresence, omnipotence, etc, etc, etc. To make a cosmological argument for the existence of God is a completely different intellectual undertaking.

    Oh, my Uncaused Cause !!!

    Davey, me boy, at least theists are going around doing intellectual undertakings. You could start trying it sometime, and see how much fun it is. I fear your, ummm, presentations, have already gone to the undertaker's.

    Anonymous has responded in part to your point. I would simply add that you cannot undertake a valid claim to an intelligible point of view about what specifically is wrong with theistic arguments for the existence of God without actually having a grasp of what the theists are saying. Nobody here is in the least interested in whether the universe had a beginning, it is simply not relevant to the theists' argument. It's hardly fair to even call your discussion so much poking holes in straw men, as rather blindfolded shooting arrows at targets based on hearsay you've heard about where they used to be located at some point in the distant past - but the targets are prime numbers. Bit of a category mistake, ain't it?

    ReplyDelete
  40. Would a B-Theory of time, as seems implied by relativity (as I understand it) have a significant impact on the potency-act distinction?
    Would not space-time, when envisaged in that fashion, mean that the universe was not in motion and ever changing, that things do not become other things, and therefore Aquinas' premises for his first cause be in difficulty?
    If the above is (remotely) correct, wouldn't the universe be a candidate for being "pure act" and therefore Aquinas' first cause?

    ReplyDelete
  41. explain the essence of photons since I'm very much in the dark on that question

    I've already done that here, or at least quoted Feynman who does it for us. Tests of this explanation show it to be accurate to nearly one part in a billion billion—if you or anyone else has an idea that can explain reality to one part in a billion billion billion, please let us all know about it.

    And I've already responded to the question of why QFT violates causality here, or at least I quoted Feynman who does that for us. But in case anyone missed Feynman's point, I'll paraphrase it with a counterfactual explanation invoking causality to illustrate the mistake of applying causality to QFT.

    Consider the phenomenon of electron-electron repulsion. This is caused by the exchange of a photon between the two electrons, as illustrated in this Feynman diagram (ignore their labels: straight lines are the electrons, wiggly line is the photon). The photon pops just into existence, so what caused it? Well, there are only three particles in this picture, so if the photon was caused by something, it must have been caused one of the two electrons. The Feynman diagram clearly shows that the photon must have been caused by the electron on the left, because we see this electron emit the photon, and then a short time later (up direction in the diagram) the electron on the right absorbs it. So the LHS electron caused the photon. Bur wait! The rules of QED are invariant to the location of all locations in space-time—in fact, the probability of an event is determined by a sum over all space and time, so we could just as well have drawn the diagram so that it shows the RHS electron emitting the photon and then a short time later the LHS electron absorbing it. So the RHS electron caused the photon. This is a contradiction. Therefore our original assumption that the photon was caused by one of the two electrons is incorrect. What caused the photon? Either nothing did, or as Feynman discusses in QED, "Probably the question has no meaning."

    This is Feynman's explanation above, and why he correctly says that QED has no causality. The photon exists, yet the rules of QFT are inconsistent with a causal explanation of its existence. Contrary to the incorrect statement by another commenter that "We never see anything 'come into existence totally uncaused from nothing'", we in fact appear to see this all the time and this is an aspect of our best explanation of the real world, good to almost one part in a billion billion.

    That extraordinary precision comes about precisely because we consider the very possibility that real things "come into existence totally uncaused from nothing" all time time. Have a look at one of the higher order terms for electron-electron repulsion. The electron-positron creation from nothing is an essential part of the explanation required to achieve the amazing experimental accuracy. So particles popping into existence are an essential part of our explanation of the world, whether they're a part of the photoelectric effect, or just pop into existence in a vacuum.

    Perhaps Feser can explain how to square Thomist causality with modern physics. What would Aquinas say caused the photon's existence in any Feynman diagram? What caused the W boson's existence?

    ReplyDelete
  42. @Steve Smith

    Dear Mr. Smith... you are makinf nonsense statments, since you are confusing concepts.

    I think it is quite WRONG to say that photons or virtual particles that come into existence have 'no cause'.

    They DO have an ontological cause: i.e. they depend on somethingelse existing for their existence, they do not just appear 'out of nothing'.

    Sure they "just pop into existence in a vacuum."... but vacuum is not 'nothing', but defitively it is SOMETHING since it has a (defined) energy level (that might even not be a ground level btw).

    So dear Mr. Smith what you are telling here, I am sorry, is a gross misunderstanding of physics or the meaning of different types of causality.

    Sure the formation of virtual particles might not have a deterministic cause, they formation might be completelu random, but still they DO have a cause, an ontological one... and not in 'metaphysical terms' only but in physics as well.

    Hence this statement is also basically nonsense:

    "Causality is meaningless in the context of the physical description of these objects, and this is the context that describes our everyday reality to almost one part in a billion billion. "

    HERE you are not really asking 'what is the cause of X' but rather 'what is the mechanistic process that leads to event X'

    The (ontological) cause of certain events, such as the formation of virtual particles or a W-boson is the existence of something actual (and pre-existent) such as a vacuum energy or a quark.

    What the 'mechanism' of formation of such events (ie how from a vacuum energy we get a virtual particle pair) is a different type of question and regards a very different type of causality that Prof. Feser is talkiing about.
    ------------


    CONCLUSION:

    It is NOT true that in the quantum regime (no matteer how small you go) particles or evens come into existence 'out of nothing' or 'without a cause'.

    There is always an ontological cause, I dare to say, even if there is no accidental or deterministic cause.

    ReplyDelete
  43. Steve Smith said...
    Feser's version of the cosmological argument is provably wrong...


    Unfortunately you did not disprove anything... you just proved one thing: you do not understand physics correctly.

    ReplyDelete
  44. Steve,

    For a Thomistic account of quantum mechanics, you might want to check out the work of Wolfgang Smith, in particular his book The Quantum Enigma. I haven't read this book yet, so I can't say whether or not I agree with it, but it does appear that it would address your questions.

    And I agree with previous commenter that you are confusing causation with efficient causation. Please familiarize yourself with the terminology that Thomists use before posting on a Thomism combox, otherwise you are just talking past everybody.

    ReplyDelete
  45. My comments are drawing very inconsistent responses: apparently my comments reveal that either I don't understand causality, physics, or ontology.

    But it's not about me, and I wonder why is no one is able to answer these simple questions? What caused the photon's existence in these two Feynman diagrams? What caused the W boson's existence?

    Electron repulsion
    Beta decay

    These should be easy questions to answer using simple, straightforward language for those claiming a grasp of causality, physics, and ontology.

    What's the answer? What caused the photon?

    ReplyDelete
  46. Indeed 'causality' is a term far too general to be taken so casually.

    Sure some "efficient causes" elude us and might be completely stochastic in origin (such as the formation of a W-boson in beta decay).
    HOWEVER there IS a cause because the W-Boson come from a process of a quark changing its 'flavor' from 'down' to ' up'.
    Now (assuming we knew very little of the weak force and electroweak theory) we still know:
    no quark, no W-boson
    but even MORE FUNDAMENTALLY: no down quark, no event of changing flavor from down to up, since for that physical event to occur there must exist a down quark that undergoes the change.
    So, SURE! To inquire about the 'efficient causation' might be meaningless (and that is what Feymann is saying, since as physicists one looks only at efficient causation usually), but 'ontological causation' is not meaningless at all!
    We know that there IS some thing as 'beta decay' and it's (ontological) rests the very existence of that down quark that changes, for whatever reason or even no reason at all if you will, into an up quark (no down quark, no change from down to up... obviously!)
    So the fact that there is a mechanism or ‘reason’ for the formation of the W-boson, or for the change of the quark’s flavor is irrelevant, ontologically speaking.

    Beta-decay occurs because there is something ACTUAL as a ‘down quark’ and something POTENTIAL as an ‘up quark’ (and W-Boson). Now that ‘potential quark or boson’ will NOT come into existence with no cause, but it is actualized by something: the Down quark.

    Now for THIS TYPE of causality it is irrelevant if the down quark has a ‘reason’ to change into an up quark or if that change is caused by something else or it is inherently in the powers of the Down quark itself.
    That’s a question of **Efficient** causation, which might or might not be meaningless.
    The same goes for virtual particles that form in vacuum and other stochastic processes.

    In essence it can be treated as a “chicken and egg” question: it is meaningless to ask which came first, but it illogical to think that one ‘just popped into existence from nothing at all’ (and NO serious physicist would make such a claim, unless he things something like gravity is ‘nothing’), since one begets the other, regardless which one came ‘first’ (hence the ONTOLOGICAL causation still has meaning even if time would have no meaning, as sometimes it happens in physics).

    ReplyDelete
  47. What's the answer? What caused the photon?


    Again: that regards EFFICIENT causation.

    The answer might exist or might not exist.

    Many scientists are trying to understand particle physics, hence they must think it exists.


    REGARDLESS if there is or is not an answer regarding efficient causation, ontologically the W-boson is caused by the change in flavor in the quark.

    The W-boson will not just 'pop into existence' if there is no such event in beta-decay.

    Not will a nuon or an apple appear instead of the W-boson (this hints at Final Causality...).

    The same goes for a photon.

    It still has an ontological cause, even if there is no efficient one.

    I do not think Feyman would say: bosons and photons appear out of nothing just like that.

    If there is no 'down quark' there will be no beta-decay out 'of nothing'.

    Also, as stated before, even vacuum is not nothingm but something in itself which can be studies and defined.

    ReplyDelete
  48. Steve:

    You keep asking people to explain what caused the particle decay, and because no-one can explain it, you conclude that there is no cause. I think that this is erroneous, because our inability to explain something does not imply that it does not exist. This would be an example of the argument from ignorance.

    The best response to this is that we do not know what causes the particle decay to occur, according to current models of physics. However, it does not follow that therefore there are no such causes. The fact that these models are incredibly predictive and useful does not mean that they have captured the sum total of reality at that level. After all, the map is not the territory, and all models inevitably miss something in reality by their abstract nature, and I would imagine that quantum theory is no different.

    And other people have explained to you that to claim that an event is uncaused would mean that it happened from absolutely nothing, i.e. ex nihilo. The fact that even in a vacuum, there is still zero point energy, and thus something, means that any quantum event necessarily has causal antecedents, because there are actual real entities and energies always present, even if we cannot specify them to our content at this time.

    ReplyDelete
  49. Steven:

    >> The rules of QED are invariant to the location of all locations in space-time—in fact, the probability of an event is determined by a sum over all space and time …

    Perhaps they are invariant to all locations in space-time, because in every location, there is the same baseline condition that affects all particles present therein in the same way? For example, the presence of zero point energy, the creation and destruction of real and virtual particles, which is present everywhere, as far as I understand, may somehow contribute to the causes of the decay in question. And in the case, the causality is certainly not ex nihilo at all, but from the background energy conditions that are invariant throughout space-time.

    Any thoughts?

    ReplyDelete
  50. Steven:

    >> we could just as well have drawn the diagram so that it shows the RHS electron emitting the photon and then a short time later the LHS electron absorbing it. So the RHS electron caused the photon.

    I admit, this part I just don’t understand.

    It seems equivalent to saying that if there is a pistol on the left side and a pistol on the right side, then the fact that a bullet could come out of either one means that there is a contradiction. It seems that there are two different scenarios here. In the first, the bullet comes out of the left pistol, and in the second, the bullet comes out of the right pistol. A contradiction would happen if the same bullet came out of the both pistols at the same time.

    In your example, a photon could be emitted by either the left or the right electron, and then absorbed by the opposite electron. A contradiction would certainly happen if the same electron was emitted by both the left and the right electron at the same time, but that does not seem to follow. Just because the equations can be applied equally to the left and to the right electron does not mean that the actual emission of an electron necessarily occurs from both electrons at the same time.

    In my pistol analogy, the same laws of physics apply to the propulsion of a bullet from a pistol, but it does not follow that a contradiction occurs. I would think a similar line of reasoning would apply to your example.

    Any thoughts?

    And one more thing, instead of saying "nothing caused the decay", an even more accurate proposition would be that "nothing IN THE MODEL caused the decay". However, it is pretty clear that "nothing in the model" does not necessarily mean "nothing in reality". There are many accurate models that exclude existing entities, after all.

    ReplyDelete
  51. dguller:

    "The best response to this is that we do not know what causes the particle decay to occur, according to current models of physics. However, it does not follow that therefore there are no such causes."

    This seems correct, of course. And the fact that physicists continue to investigate this phenomena, attempting to explain how their predictions are so accurate means that they are attempting continually to overcome this (possibly temporary) epistemic limitation.

    What Steve doesn't seem to understand is that to admit positively that some thing has no cause, that it just begins to exist ex nihilo, is to say the whole search for a theory to explain QED is a fruitless endeavor. A bit dogmatic, and anti-scientific, eh?

    ReplyDelete
  52. ontologically the W-boson is caused by the change in flavor in the quark.

    No. This reveals a failure to understand to rules of QFT, and represents the same error in ascribing causality to couplings in Feynman diagrams that I paraphrased above. You could just as well say that the W boson changed the quark's quantum state which is what happens in inverse beta decay. I'm not going to keep going round-and-round on this—QFT is a noncausal theory because events can happen anywhere in space-time.

    You keep asking people to explain what caused the particle decay, and because no-one can explain it, you conclude that there is no cause.

    No. I merely observe that no cause is necessary to explain very accurately all known physical phenomenon, and that ascribing cause in the context of our best explanation of the world is nonsensical.

    ReplyDelete
  53. The community against what Steve is presenting needs to put their heads together and ask very simple philosophical questions, because he ain't gettin' it.

    ReplyDelete
  54. instead of saying "nothing caused the decay", an even more accurate proposition would be that "nothing IN THE MODEL caused the decay".

    Yes, that goes without saying because it's the model that's used to describe everything. If there was anything else in the model, we could discuss it.

    You could try to add something like Aquinas's angels to the model, but Bell says that angels or anything else would fail to satisfy the prediction of QM, so we can rule out the angels too.

    The model is obviously incomplete in several famous ways and must be modified to account for known difficulties, but if these can ever be surmounted, any subsequent theories are necessarily refinements on the QM picture presented here. If there are any Thomist ideas that can be used to describe nature to one part in a billion billion billion, please let everyone know.

    to admit positively that some thing has no cause, that it just begins to exist ex nihilo, is to say the whole search for a theory to explain QED is a fruitless endeavor.

    No. This is nonsense.

    ReplyDelete
  55. Steve:

    I like your automatic pat gainsaying:

    No. This is nonsense.

    I think I'll write that up on the board like Mr. Hand, so that all us dumb students here can admire it.

    ReplyDelete
  56. Steve Smith, tell us what you think a "cause" is. This discussion isn't going to go anywhere until you play that card.

    ReplyDelete
  57. tell us what you think a "cause" is

    That's what I keep saying: I don't know what "cause" means in the context of QFT, and neither does anyone else. All I know how to do is compute probabilities of events. "Cause" isn't part of that calculation.

    Rather, it's incumbent on the people making physical statements about "cause" to say what they mean in the context of known physics.

    Feser said in his post that "what comes into existence has a cause". Here's a very basic example of some stuff that "comes into existence": What caused the photon's existence in these two Feynman diagrams? What caused the W boson's existence?

    Electron repulsion
    Beta decay

    What does Feser mean when he says that the photon "has a cause"? What caused it?

    ReplyDelete
  58. What caused the photon's existence in these two Feynman diagrams? What caused the W boson's existence?

    Either we know the answer, or we do not.

    If we do not know, it does not support your definitive conclusion that the W boson comes into existence ex nihilo. This conclusion is required for you to defeat that premise of the CA. Can you honestly say that is the unavoidable conclusion supplied by QED?

    ReplyDelete
  59. If we do not know, it does not support your definitive conclusion that the W boson comes into existence ex nihilo. This conclusion is required for you to defeat that premise of the CA. Can you honestly say that is the unavoidable conclusion supplied by QED?

    I honestly don't believe that you've been reading what I or Feynman wrote. A few posts up: "I merely observe that no cause is necessary to explain very accurately all known physical phenomenon, and that ascribing cause in the context of our best explanation of the world is nonsensical." Ex nihilio creation of particles is not a definitive conclusion, but it is contingent on the available evidence.

    Can anyone—anyone?!—say what Feser means when he says that a photon appearing in a Feynman diagram "has a cause"?

    ReplyDelete
  60. If you didn't have some prior notion about what a "cause" is, you wouldn't be able tell whether it applied to QFT or not. So what you really meant to say was: "I have a notion of "cause", and I don't see how it can be rendered intelligible within the context of QFT." Fair enough. Tell us what your prior notion of "cause" is, and explain why it does not apply at the quantum level. Then we can move forward.

    ReplyDelete
  61. Tell us what your prior notion of "cause" is, and explain why it does not apply at the quantum level.

    You have have it backwards. Feser made a statement about physics: "what comes into existence has a cause". What is relevant here is what Feser means by "cause". In the basic physical example of electric repulsion, what does Feser mean when he says that a photon appearing in the Feynman diagram "has a cause"?

    ReplyDelete
  62. >You have have it backwards. Feser made a statement about physics:

    Feser is a philosopher making a philosophical metaphysical argument.

    Do you not get that?

    ReplyDelete
  63. My spider senses tell me Steve Smith is trying to dodge the question.

    ReplyDelete
  64. Feser is a philosopher making a philosophical metaphysical argument.

    Forgive me for interpreting Feser's statement about existence to have some physical relevance in the real world.

    Shall I conclude that what Feser writes is irrelevant to what really happens in this world? If there's no actual physical consequences to what Feser writes, then I have no further comment on it, except to necessarily conclude that Feser's version of the cosmological argument is also irrelevant in the real world.

    ReplyDelete
  65. >Forgive me for interpreting Feser's statement about existence to have some physical relevance in the real world.

    The assumption that physics alone constitutes a description of the real world is a metaphysical position. Do you really not understand that?

    Really?

    ReplyDelete
  66. Now can you or can you not answer the question put to you?

    Tell us what your prior notion of "cause" is, and explain why it does not apply at the quantum level.

    Dancing around the question like so many angels on a pin isn't working for ya.

    ReplyDelete
  67. The assumption that physics alone constitutes a description of the real world is a metaphysical position.

    Actually, it's an empirical position whose accuracy is currently tested to a little more than one part in a billion billion.

    Do have any evidence that there is something other than physics required to describe the universe? What is that evidence?

    And please clarify my questions above, because I'd be surprised if Feser or others thought that your characterization of Feser's statements is fair: when Feser wrote "what comes into existence has a cause", does this statement on existence and cause have any relevancy to the real world or not?

    ReplyDelete
  68. Mine is the more basic question. You want to know what caused the photon. I want to know what you think a "cause" is in the first place, so I can understand why you think the photon's existence must be "uncaused." Tell me what a "cause" is so that I will be able to understand why QFT is "noncausal."

    ReplyDelete
  69. >Actually, it's an empirical position whose accuracy is currently tested to a little more than one part in a billion billion.

    The assumption that empiricism alone constitutes a description of the real world is also a metaphysical position.

    >Do have any evidence that there is something other than physics required to describe the universe? What is that evidence?

    By "evidence" you clearly mean empirical or scientific not philosophical I take it?

    These might help you get on the same page as the rest of us.

    Blinded by Scientism:
    http://www.thepublicdiscourse.com/2010/03/1174

    Part 2
    http://www.thepublicdiscourse.com/2010/03/1184

    We do philosophy here not scientism bro unless you would like to attempt a philosophical defense of scientism, empiricism or Positivism in general.

    Though I should warn you AJ Flew at the height of his Atheism gave up such endeavors in the early 50's because the concepts where whole self-referential.

    I take it you can't define what you mean by "cause" but your other statements allow me to guess it is likely you are talking about "efficient causes" not formal, material, or final causes.

    Cheers.

    ReplyDelete
  70. Tell me what a "cause" is so that I will be able to understand why QFT is "noncausal."

    As I said, I don't know what "cause" is in QFT, so I can't tell you what I think it means because "cause" isn't part of QFT. That's why QFT is noncausal—it doesn't depend on any concept of "cause".

    However, Feser wrote that "what comes into existence has a cause", so Feser thinks he knows what "cause" means. What does Feser mean by "cause"? Is it relevant to the real world?

    ReplyDelete
  71. >As I said, I don't know what "cause" is in QFT,

    Then any claim on your part QFT overthrows any version of the CA is meaningless.

    >so I can't tell you what I think it means because "cause" isn't part of QFT.

    I'm still guessing you mean strictly efficient cause here you just know it.

    >That's why QFT is noncausal—it doesn't depend on any concept of "cause".

    I think you are equivocating "noncausal" with "uncaused". Nice slight of hand. But I don't think you do it on purpose.

    ReplyDelete
  72. Ben

    What you are dealing with here is "McAtheism". The product on offer is a feeling of superiority bought at the price of a few soundbites.
    There is no serious effort on the part of the sceptic to enter into any dialogue with the Theist. It would take too much intellectual effort to rethink fundamental commitments.
    So your interrogators are trying to trip you up with a little spin.
    They're really no better than engineers who take their views on economics from reading Scott Adams.

    Although Scott Adams did sum their mindset up quite neatly...

    http://dilbert.com/strips/comic/1997-12-22/

    http://dilbert.com/strips/comic/1997-12-23/

    But I wish you Godspeed nevertheless!

    Graham

    ReplyDelete
  73. Saying "we don't know what causes x" is not the same thing as "x is not caused." Got that at least?

    To even be able to say QFT is a "noncausal" theory implies you DO have a concept of "cause," even if it is imperfect or undefined. It's the only way "noncausal" can have any meaning, as the negation of some concept of "cause." So, we want to know which concept it is.

    ReplyDelete
  74. A wonderful McAtheist McNugget

    The smarter you are, the harder you have to work to believe.

    It is incredible - but many well educated, intelligent atheists believe that this sort of quip counts for something...

    ReplyDelete
  75. @Havok,

    Would a B-Theory of time, as seems implied by relativity (as I understand it) have a significant impact on the potency-act distinction?
    Would not space-time, when envisaged in that fashion, mean that the universe was not in motion and ever changing, that things do not become other things, and therefore Aquinas' premises for his first cause be in difficulty?
    If the above is (remotely) correct, wouldn't the universe be a candidate for being "pure act" and therefore Aquinas' first cause?


    No, since under the hypothesis, there is no change, so no need to account for change, so no need to distinguish between actual and potential, so no sense in describing the universe as "pure act", as it doesn't act. Of course, such a metaphysical position would need to account for the existence of "awareness of change", and in doing so will likely reopen the whole can of worms -- that is, one is no longer talking about "the nature of reality", but instead about "the nature of our illusion", and have the same questions.

    ReplyDelete
  76. If you don't know what a "cause" is, then you don't know whether the Cosmological argument is sound or not. Nothing wrong with that, but it isn't what you said earlier. Earlier, you talked a bunch of trash about how QFT makes "mincemeat" out of the cosmological argument and how we all just need to learn some physics. It is now apparent that you can't back up any of your trash talk, because by your own admission you have no clue what a "cause" is. And yet, despite having no clue what a "cause" is, you are absolutely sure that there aren't any "causes" in the quantum domain. Big hat, no cattle.

    ReplyDelete
  77. Mr. Veale,

    I feel the strong dark temptation to mock Steve Smith without pitty or mercy but I will control my natural Sith Lord impulses and see what unfolds.

    Maybe it will convince him of the necessity of philosophy if nothing else?

    ReplyDelete
  78. Josh is correct.

    To even be able to say QFT is a "noncausal" theory implies you DO have a concept of "cause," even if it is imperfect or undefined. It's the only way "noncausal" can have any meaning, as the negation of some concept of "cause." So, we want to know which concept it is?

    I can't help but notice it is a common New Atheist trick to keep shifting the burden of proof. That can only take you so far.

    Feser due to his Aquinas and Aristotelian commitments will define "cause" as either "efficient, material, formal or Final" and there are I believe subdivisions of that.

    It's not a real mystery to those who learn the Aristotelian terminology.

    But at this point Steve Smith terms are the mystery.

    ReplyDelete
  79. If you don't know what a "cause" is, then you don't know whether the Cosmological argument is sound or not. Nothing wrong with that, but it isn't what you said earlier. Earlier, you talked a bunch of trash about how QFT makes "mincemeat" out of the cosmological argument and how we all just need to learn some physics.

    Forgive me for assuming that Feser was referring to the real physical world when he wrote "what comes into existence has a cause".

    If what Feser writes isn't relevant to the physical world, then what Feser writes isn't relevant to QFT. And neither is Feser's cosmological argument relevant to to the real world.

    If Feser intends his statement to apply to the real world, then QFT is directly relevant, and Feser must tells us what he means by "cause", because there is no such concept used in QFT.

    Can anyone here answer a few simple questions with simple answers?

    Is Feser's statement "what comes into existence has a cause" relevant to the real world? What does Feser mean by "cause"?

    ReplyDelete
  80. To even be able to say QFT is a "noncausal" theory implies you DO have a concept of "cause," even if it is imperfect or undefined.

    No. QFT is also a "non-angel" theory in that it does not depend on Aquinas's angels, whatever they are.

    All that is necessary is to define all the ingredients of the theory. Anything not in that list of ingredients makes the theory non-whatever isn't on that list: non-angel, non-god, non-causal, … you get the idea. QFT is none of these things.

    What does Feser mean by "cause"? Is it relevant to the real world?

    ReplyDelete
  81. >No. QFT is also a "non-angel" theory in that it does not depend on Aquinas's angels, whatever they are.

    Angels are for Aquinas Substantial Forms. Angels can't create ex nihilo according to Aquinas either.

    Your ignorant mockery is altogether tedious.

    >Can anyone here answer a few simple questions with simple answers?

    Clearly not you.

    ReplyDelete
  82. The problem here is we are all trying to argue philosophy and Steve Smith refuse to do so.

    Yet he make as bunch of implicit philosophical claims "Physics and Empirical data alone describe reality etc" and won't admit doing so.

    You know Steve you really should learn philosophy. The idea Science alone describes the "real world" is not a scientific claim. It's a philosophical one.

    Live with it. Peace to you.

    ReplyDelete
  83. Ok Stevo, since you seem determined to wallow in sophistry, how about a different angle?

    "All that is necessary is to define all the ingredients of the theory. Anything not in that list of ingredients makes the theory non-whatever isn't on that list: non-angel, non-god, non-causal, … you get the idea. QFT is none of these things."

    Let me try to understand this better. My "theory to describe x" does not include the terms "a, b, or c," therefore, insofar as the theory is understood, then it can be predicated of the theory that it is "non-a, non-b, etc." Is that correct?

    It would seem that it does not follow, in my estimation. Under this explanation, then, the "theory of x" isn't about "a, b, or c" at all. Meaning you cannot say anything about "a, b, or c" from the "theory of x." Which in turn means you cannot use QFT to defeat the CA's understanding of causation. They literally have nothing to say about each other. They don't touch.

    ReplyDelete
  84. I am confused by all of this because it appears to be going in circles. As a true novice, I have been trying to follow the conversation. I see that there is a word used frequently, noncausal. The "non" prefix would imply that the word is not the root word, in this case cause. The question that keeps being glossed over here is what does cause mean. If some thing is nonworking, I first need to know how working is defined. If I do not know the definition of working in this case, I cannot conclude that something is "not" doing something that is an unknown. So, the question remains, what is cause in QFT?

    ReplyDelete
  85. BTW, Steve, quit asking what Feser's definition of "cause" is. It's readily available to anyone who reads his work that he subscribes to Aristotle's idea of four essential causes. So please quit asking.

    ReplyDelete
  86. Steve,

    "Is Feser's statement "what comes into existence has a cause" relevant to the real world? What does Feser mean by "cause"?"

    First question. Yes. But it does not fall in the domain of physics. Rather physics will have to presuppose what we are talking about here. The talk about causality and change are so general that any possible theory within physics must fall within its bounds.

    Second question. Look into the Aristotelian notion of cause. There are four. Aquinas might add another sense if memory serves.

    Bottom line. You have to fit in another area of possible knowledge within your view of reality. It's not science. It's not simply logic or axiomatic systems devoid from the world. It's a part of metaphysics (hopefully done properly).

    ReplyDelete
  87. My "theory to describe x" does not include the terms "a, b, or c," therefore, insofar as the theory is understood, then it can be predicated of the theory that it is "non-a, non-b, etc." … It would seem that it does not follow, in my estimation. Under this explanation, then, the "theory of x" isn't about "a, b, or c" at all. Meaning you cannot say anything about "a, b, or c" from the "theory of x."

    That's wrong, and the counterexamples should be obvious. QED doesn't depend on "electrical charge", so QED is a non-"electrical charge" theory. Yet "electrical charge" is one of the immediate consequences of QED.

    Similarly, QED is a non-causal theory in that it doesn't depend on any notion of causality, such as such-and-such happens first, then this-and-that happens as a consequence afterwards. Yet various notions of causality are a consequence of the probability predictions made by QED.

    That's why, if Feser's statement has any relevance in the real world, it's necessary for Feser to say simply and explicitly what he means by "cause".

    ReplyDelete
  88. what is cause in QFT?

    How many times must it be said?! QFT doesn't depend on any notion of "cause". There is no "cause" in QFT.

    ReplyDelete
  89. … Feser's definition of "cause" is. It's readily available to anyone who reads his work that he subscribes to Aristotle's idea of four essential causes.

    Which one of Aristotle's four causes is Feser using when he writes, "what comes into existence has a cause"? All of them? Any of them? Which of these four applies to the photon in the Feynman diagram for electric repulsion?

    ReplyDelete
  90. The talk about causality and change are so general that any possible theory within physics must fall within its bounds.

    No. The talk about causality and change are so general that they encompass contradictory speculations about reality.

    Physics is the only thing that will tell you which hypotheses are correct, and which are not. The metaphysics must conform to the physics if it can be said to have any relevance in the real world.

    ReplyDelete
  91. I must sign off. I wish that we could have found at least of toehold of common ground. I'd encourage everyone interested to read Feynman's QED for some great insight about how the ideas discussed here will either work or fail in the real world.

    ReplyDelete
  92. Steve

    "No. The talk about causality and change are so general that they encompass contradictory speculations about reality."

    True, but only if the "speculations" you are talking about are of the scientific variety. Otherwise, what you say doesn't make sense.

    "Physics is the only thing that will tell you which hypotheses are correct, and which are not."

    And this is the problem you are having. As long as you refuse to let metaphysics (which can use empirical observations and reach necessary conclusions by deduction, and not just hypotheses) have any room at the table, you will not understand Aquinas' arguments.

    Again, please realize that your above statement is a philosophical claim. It has to be defended and not assumed if you want to hang out here.

    We could speak further about causes (and by all means please keep discussing it), but this is a much more fundamental issue.

    ReplyDelete
  93. So, then, I understand you to say that QFT doesn't make use of causality per se, but has implications for causality per se? How exactly does QFT give rise to theories of causation then?

    BTW, in following that link, I see a bunch of theories that attempt to explain what you are talking about in terms of cause and effect. Why the hell would they need to do that if cause has nothing to do with this? It seems that those theories are committed precisely to explaining the antecedent conditions that would give rise to such things as the W boson's existence. How am I wrong in this?

    ReplyDelete
  94. Steve said: As I said, I don't know what "cause" is in QFT, so I can't tell you what I think it means because "cause" isn't part of QFT. That's why QFT is noncausal—it doesn't depend on any concept of "cause".


    Oh, I think I see now: Steve isn't saying there are no causes, he's just saying that there's no variable standing for "cause" in Quantum Field Theory. But of course lots of things are not mentioned in the QM toolkit as used by physicists -- Tchaikovsky or the White House, for example. Doesn't mean they don't exist.

    As for what "causality" is, Feser means material, formal, efficient, or final explanations of a thing's being. And such causality applies to everything that exists, or ... it wouldn't be a being. (Well, with one and only one possible exception. Or perhaps I should say actual exception.) And technically, QFT is full of formal and final causality, by its very nature. (It is irrelevant that it may not say so explicitly, any more than QM's reliance on calculations means you're going to find in the formulas an "m" standing for "mathematics".)

    ReplyDelete
  95. "Oh, I think I see now: Steve isn't saying there are no causes, he's just saying that there's no variable standing for "cause" in Quantum Field Theory."

    If that's true, then come back Steve! There's no problem here! Only if you are saying that there is NO cause of some thing's coming into existence is there an issue. Very good there, anonymous. I've been hopelessly confused the past few posts.

    ReplyDelete
  96. Would I be right in saying the corrected argument presented here is saying:

    1. That everything that exists has a cause. But not that everything has a cause, nor that god was the 'first' cause in terms of going backwards in time.

    2. This cause of existence is god.

    3. God was uncaused?

    If I am correct, if perhaps overly-simplifying, then doesn't the logic break down when we ask if god exists wasn't he caused? But he was uncaused.

    I'm an atheist who is really trying to get my head around theology, perhaps not enough, but it's things like that I can't get past.

    Do the points Feser mentions, but doesn't go in to, argue why god was uncaused, or why he can't be assigned 'existence'? I freely admit I may be missing things that aren't in this argument.

    ReplyDelete
  97. Matt Dickinson:

    From the post:
    "1. The argument does NOT rest on the premise that “Everything has a cause.”"

    "What defenders of the cosmological argument do say is that what comes into existence has a cause, or that what is contingent has a cause. These claims are as different from “Everything has a cause” as “Whatever has color is extended” is different from “Everything is extended.”"


    So, reading that, do you have any more questions?

    ReplyDelete
  98. Josh said: "Only if you are saying that there is NO cause of some thing's coming into existence is there an issue. Very good there, anonymous. I've been hopelessly confused the past few posts."

    Yeah, in some sense you could say that "causality" is superfluous to quantum mechanics. Maybe there's a cause there, maybe not, but QFT isn't going to tell you, any more than it will tell you whether the White House exists, or Obama is president. So it's "non-causal" or "non-presidental", in Steve's terms. A better term would be "causality neutral", since the point is the physical theory is agnostic about it.

    Of course, it's not neutral at all in the Thomistic sense, because the physics obviously relies on things having certain well-defined properties and acting in certain well-defined ways. But Steve is no doubt thinking of "efficient causality", in fact, "physical efficient causality", about which the science is silent. And we don't need physical efficient causation for the science to work, or the philosophy. Hidden variables are a bit of a red herring. If the physics works without them (and that seems unlikely to change), then so does the metaphyiscs, it's not like we need to posit some "metaphysical particles" to do the job. No need to multiply causes beyond what is necessary.

    ReplyDelete
  99. "A better term would be "causality neutral", since the point is the physical theory is agnostic about it."

    That's precisely what I was taking from what Steve was saying, but I don't think he got that, ironically. This is a much more elegant way of saying it, I think.

    ReplyDelete
  100. "But Steve is no doubt thinking of "efficient causality", in fact, "physical efficient causality", about which the science is silent."

    Getting him to admit that would be like trying to draw blood from a stone, I think.

    ReplyDelete
  101. @Josh
    That was my point, apologies, perhaps I didn't express it clearly. Here's my attempt to re-word my understanding of the argument:

    1. Everything that comes into existence has a cause. But not that everything has a cause, nor that god was the 'first' cause in terms of going backwards in time.

    2. God exists [I'm pretty sure this is part of the argument]

    3. God is uncaused.

    Perhaps I'm also missing the distinction between things that COME INTO existence and god, who just exists.
    If that is the case I don't think the difference was emphasized enough. But again, it may have been my failing.

    Other than that I have no more questions, just the same one as before.

    ReplyDelete
  102. Matt:

    Slight problem there in step two...the existence of the uncaused cause is not an assumed premise, but the conclusion of the argument. Some of the more learned guys here could supply you a better syllogism to speed you along, I think.

    ReplyDelete
  103. Josh:

    I thought there was something I was missing, thanks for spotting it!

    Perhaps I should brush up on my logic, or think more before I post comments, but I'm trying to write my understanding out in my own words and get round the jargon Jerry Coyne and Edward Feser have been debating about.

    So it is the difference between something which 'happens to exist, but which could not even in principle have failed to exist'?

    So does the cosmological argument say whether the universe could be the 'something that could not in principle have had a cause, or that is purely actual, or that has a sufficient reason for its existence within itself'? As opposed to god being that something?
    This is a different argument from that discussed by Feser on whether the universe had a beginning (which Stephen Hawking as argued it doesn't from a 4-D perspective).

    If the full cosmological argument doesn't argue/discuss whether the universe is pure actuality, do other arguments cover this?

    ReplyDelete
  104. If there is no efficient cause, then doesn’t that undermine the explicability of the phenomena in question, though? In other words, if we can understand something from the formal, material and final causal perspectives, but there is no efficient causal mechanism, then can we really say that we understand it at all? I think that without an efficient causal mechanism, something really isn’t truly caused at all.

    And if Steve is right that there is no efficient cause of the particle decay involved in QED, then that would undermine the cosmological argument IMO. In other words, even if there were material, formal and final causes, without the efficient cause, the phenomena in question is essentially uncaused from my perspective. It would be like saying that someone got from point A to B, because their goal was to get from A to B, and their form was such that they are someone who likes to go from A to B, and their matter physically allowed them to move in space, but there is no efficient cause for how they got from A to B. Would you really say that this situation can be understood as caused?

    And to say that it is a metaphysical argument is irrelevant, because its key premise is empirical, and if there is a scientific theory that undermines that empirical premise, then the argument is weakened, whether it is physical or metaphysical.

    The best that can be said against his argument is that just because a scientific model does not include certain things does not necessarily mean that those things do not exist. There are a number of examples of where this is obviously true. However, if there is a model that can explain all physical phenomena without the need to postulate any efficient causes whatsoever, then this argument becomes weakened in my eyes, because then it seems that people who are desperate for those things to be real and are clinging in the face of reason rather than because of it.

    ReplyDelete
  105. Steve Smith: "what causes the creation of a photon?"

    From wiki: "photons and electrons do, somehow, move from point to point and electrons, somehow, emit and absorb photons. We do not know how these things happen, but the theory [QED] tells us about the probabilities of these things happening."

    It seems obvious then, to my simple brain, that electrons cause photons... for, without electrons, photons would not be emitted.

    For photons to be uncaused, they'd have to be emitted by nothing. Electrons are not nothing.

    ReplyDelete
  106. Matt Dickinson‬: 1. That everything that exists has a cause. But not that everything has a cause, nor that god was the 'first' cause in terms of going backwards in time. 2. This cause of existence is god. 3. God was uncaused?

    I almost thought you were saying, "1) Everything has a cause. ... 3) God doesn't have a cause" — but on a closer reading, I see you actually said, "1) Everything that exists has a cause. ... 3) God doesn't exist." Which of course, yes, makes it confusing since theists obviously claim that God is real, just that He doesn't "exist" in some sense. Which is almost right.... God doesn't "exist" the way everything else does, because God is existence. And yes, Feser does go in to those details a lot more in other places (there are a lot of good articles on this site, but if you're new to the subject you probably want an organised approach that starts from the ground up, in which case his Aquinas is a very good and easy-to-read introduction).

    ReplyDelete
  107. Matt Dickinson‬: Here's my attempt to re-word my understanding of the argument:
    1. Everything that comes into existence has a cause. But not that everything has a cause, nor that god was the 'first' cause in terms of going backwards in time.
    2. God exists [I'm pretty sure this is part of the argument]
    3. God is uncaused.


    1) Everything is either (a) caused, or (b) uncaused (simple logic)

    2) Nothing can cause itself (I can, say, cause myself to move, but really that's one part of me causing an effect in other parts. A specific effect X cannot cause itself, because if X is the effect, then by definition it doesn't exist until it gets caused. Before it's caused it doesn't exist, and if something doesn't exist, it can't do anything, let alone cause itself!)

    3) Therefore, everything that is caused is caused by something else — or rephrasing (1), everything is either caused by something or else not caused at all

    4) Could everything be caused (i.e. there is no "uncaused" thing)? No, because in any series of things wherein one causes another, which causes the next, etc., there has to be a first being which isn't caused. (If the first being itself needed to be caused like all the others, then nothing would ever happen at all — the first cannot cause itself, and there can be no other thing to cause it because it is the first.)

    [Sub-argument: what if the series of caused beings is infinite? Then there is no "first" being, won't that work? Still no: consider a train, with "caused" replaced by "pulled", and "uncaused being" replaced by "railway car that is not pulled by anything, aka the engine". Clearly a train with a finite number of cars will not move unless there is at least one engine there to pull the others. If you just have cars but no engine, there's no "unpulled puller" to start pulling one of the cars (which could then pull the next car, etc.). But clearly making the train infinitely long will not help if there is still no engine. An infinite number of cabooses does not make an engine.

    Or consider this: suppose there is an infinite series of mirrors, each reflecting an image from the previous mirror. If there is an image (e.g. a light bulb) to be seen, then somewhere light must be coming from a non-reflecting being (i.e. from the actual bulb, not just another mirror). Even if you had an infinite series of mirrors, so that you could explain the image in any one mirror by pointing to the one before it, an infinite series of mirrors cannot explain the light bulb image out of nowhere.

    Note that we are not saying that an infinite series is impossible — Aquinas quite carefully allows for such possibilities. Just that if you do have an infinite series of mirrors reflecting a bulb, then you must have a bulb as well; or if you do have an infinitely long train, you must have at least one engine for it to move.)]

    5) So there must be at least one uncaused being. (And there could be caused beings as well, because now we have the uncaused being which can cause other things.)

    That's really, really brief, of course, and more could be fleshed out — it takes Feser at least 70 pages to get that far in his book, but that should give you an idea. Then there are many more arguments to follow (e.g. how do we know there aren't more than one Uncaused Cause? Because if there were, then they would have to be different in some way (or else they're just the same being!). But there must be a cause, i.e. explanation for that difference, and that cause would contradict their both being Uncaused.) And so on and so forth. Even if you don't buy the whole system, it's very impressive how far Aquinas can go with his careful and precise arguments.

    ReplyDelete
  108. So dguller, is your question:

    Does denying the agency of one of the four causes (in this case, the efficient) entail the denial of the agency of all the causes?

    ReplyDelete
  109. Mr. Green

    Thank you. I have come across the idea that god is existence before, but couldn't work out if this subtly was part of the cosmological argument or an assumption of it. (And I'm still unclear :P)

    I'm tempted to read Feser's book, I only came across him and this blog tonight, but I was hoping I could find my answer without doing so.
    I understand it's a complex subject however!

    Does the book cover and clear-up the misinterpretations and other philosophers (Leibniz etc)?

    ReplyDelete
  110. Fire can't be the efficient cause of water turning to ice. Since the heat of fire is more likely to turn water to steam.

    OTOH fire burning a fuel that powers a refrigeration unit has causal power in turning water to ice. Just not efficient causal power.

    ReplyDelete
  111. Mr. Green

    Thank you more for your second comment, I missed it while I was replying to your first.

    I do understand infinite regression, but I thought that's what Feser was arguing the cosmological argument (CA) was not, because that surely can be put away with 'why not stop at the universe or why stop at god as the uncaused thing?'

    If the CA says god is existence rather than an uncaused thing that exists then I see how it is not open to quite the same infinite regression argument. Though I'm still not convinced 'god is existence' is any better I see how it's different.
    But how do you stop there, if god is existence then that merges into the Hindu idea of Brahmin or the Dao, which is close to deism, which is close to Einsteinian 'belief' in the laws of the universe.

    Perhaps that is going too far out of the remit of the CA, however?

    ReplyDelete
  112. Sorry, I meant Brahman 'the one supreme, universal Spirit that is the origin and support of the phenomenal universe' (as wiki puts it) not Brahmin.

    ReplyDelete
  113. Matt Dickinson,

    "If I am correct, if perhaps overly-simplifying, then doesn't the logic break down when we ask if god exists wasn't he caused? But he was uncaused."

    It's a shell game. They simply want us to assume that under one of their carefully positioned shells there is an uncaused cause. Then, big surprise, that hidden and never seen uncaused cause conveniently turns out to be their favored conception of god.

    ReplyDelete
  114. In TLS the Unmoved Mover argument is presented via a hand, stick, and stone example. (p94) The stone moves only insofar as the stick moves which moves only insofar as the hand moves. Of course, the hand moves via the arm via muscles via neurons via molecules via electromagnetism, etc. --- all simultaneously. He then asserts, "an essentially ordered series, of its nature, must have a first member." Furthermore, if there were "finally no first member of the series, there'd be no series at all in the first place, because it is only the first member which is in the strictest sense really doing or actualizing anything."

    The fault with this argument is staring him in the face: There really is no series at all in the first place Feser gets us to focus so hard on the "chain" and the "series" that we don't see the chain is a magician's illusion.

    Keep your eye on the chain, folks! Look up the arm, follow us even into the misty reality of the molecule. Forget the obvious. Forget that the human who holds the stick also stands on legs which are on a planet which is held in orbit by the sun. What caused the sun to shine today, at this very moment? Here's some fun. Why don't we follow that chain as well? In fact, we could find countless "chains" to investigate. All of these chains intersect with that stick and stone. All are just as deeply involved in a separate but equal "series" of causation. And when we're really deep, no matter how sub-atomic we go, we still have to explain why that nasty neighboring sub-atomic force is tugging away at us, probably in the wrong direction. Why does it mess with our final plan?

    So why does Feser look at that one simple chain? Why does he stop at an arbitrary "First Cause"? Because he has found an answer he likes. It agrees with his ideology. His fable is presented as "reason" which is then twisted to conform to his personal and highly subjective Final Cause. That's the only Final Cause in this scenario.

    ReplyDelete
  115. Djindra, mangling arguments in the service of wacky political motivation.

    Keep on truckin', bro!

    To the rest, great comments. Nice to see the cosmological argument as classical theism presents it so ably presented and defended.

    ReplyDelete
  116. Donny boy Jindra is our little court jester, except he can't even get that role right

    ReplyDelete
  117. I am new to posting here as well, although I've followed the blog for some time, have read TLS and I'm halfway through with Aquinas. Thank you to everyone here - I've read every one of these 300+ posts. Writing this post, originally to Steve Smith, has been a good learning opportunity. Although Steve Smith has moved on, I'll post it anyway in case someone has a comment that either or I others would benefit from.

    Steve Smith, I don't see that you responded to my posting of Dr. Feser's explanation of efficient causation and its limitations, nor to several of the posts making the same points. Maybe efficient causation would be clarified by considering final causation in contrast.

    I'll point you back to Dr. Feser. All of the following quotes come from Aquinas, the section on the Fifth Way, beginning on p. 110. Please note that with the Fifth Way, we are leaving the cosmological argument (which is found in the First, Second and Third Way) and embarking on the teleological argument. Thus, what follows is a description of "final cause" and not "efficient cause." This section is couched mostly in terms of evolution as a rebuttal to those who try to reduce Aquinas' argument to the "design argument" of Paley. (If you make that mistake, then the Thomists here will carve you into little pieces, perhaps even to "one part in a billion billion.") The explanation about final causation covers inorganic processes as well.

    "...as Garrigou-Lagrange points out, even a simple physical phenomenon like the attraction between two particles would suffice for his (Aquinas') purposes. What he is saying is rather that it is impossible that every apparent causal regularity can be attributed to chance, for chance itself presupposes causal regularity."

    "Given what Aquinas says about chance (and as Garrigou-Lagrange has also pointed out), it is a mistake to think that the "principle of finality" on which Aquinas' argument rests says that "everything has a final cause" (just as we have seen it's a mistake to assume that the principle of causality says that "everything has an efficient cause"). For not "everything" does have a final cause, given the existence of chance events. What Aquinas actually says, as we have seen, is that every agent has a final cause; that is to say, that everything that serves as an efficient cause "points to" or is "directed at" some specific effect or range of effects as its natural end. This is why it is silly to ask (as is sometimes done) "What is the purpose of a mountain range?" or "What is the purpose of an asteroid?" as if such questions must be an embarrassment to any Aristotelian. Aquinas would be happy to allow that such might turn out to serve no "purpose," in the sense of being accidental byproducts of convergent natural processes...He would insist, however, and quite plausibly, that such natural processes embody patterns of efficient causation that are themselves intelligible only in terms of final causation. And precisely for that reason, to the extent that biological processes like evolution manifest causal regularities, they if anything only support the Fifth Way rather than undermine it. For as with mountains, asteroids, and the like, even if it should turn out that animal species are the accidental byproducts of various convergent impersonal causal processes, the existence of those evolutionary processes themselves would require explanation in terms of final causes.

    Therefore, "accidental byproducts of various convergent impersonal causal processes" can be considered without calling on the angels to pinch hit.

    ReplyDelete
  118. djindra, those are the sort of arguments I am wont to make the problem I have with them is not to do with them in themselves, but that I worry I am starting with an assumption myself. The assumption that theology is misdirection based on pre-decided results.

    In trying not to fall into the same trap myself I'm trying to keep within the cosmological argument and work out what it is I don't understand (according to Feser's title).

    If the misunderstanding as Feser claims is that god isn't the first/prior cause to the universe, but self-caused existence itself, I still don't see why the universe can't be self-caused existence itself. Many thinkers in Eastern thought and more recently philosophers regarding the claims of physics have imagined the universe as this.

    It seems no matter what way you cut it Occam's razor can still be brought into effect.

    Perhaps I am making the fallacy that Feser calls out when he says the cosmological argument doesn't explain why god has other qualities that would separate from the universe (all-loving etc)?

    ReplyDelete
  119. To put it more clearly, I think theology works with preconceptions (that god exists and what his qualities are), but I worry that this thought of mine is a preconception, so I don't want to be hypocritical. Therefore I am trying to be fair here, confining myself to the cosmological argument and it's misinterpretations (i.e. Feser's topics).

    But however god is defined in the cosmological argument (excluding personal qualities, benevolence, awareness etc) I do not understand why the same cannot be said of the universe and the words are interchangeable. The universe is 'something that could not in principle have had a cause, or that is purely actual, or that has a sufficient reason for its existence within itself'

    ReplyDelete
  120. "A better term would be "causality neutral", since the point is the physical theory is agnostic about it." … This is a much more elegant way of saying it, I think.

    No. As already stated, QFT is the physical source for our notions of causality because of the way it computes probabilities. If that we're not so, none of us could peck at our keyboards and send messages into teh internets.

    Only if you are saying that there is NO cause of some thing's coming into existence is there an issue.

    Please re-read Feynman's account of physics, or my attempt to paraphrase it. The notion of physical causality is nonsensical in quantum field theories because particles can take any continuous path through space-time. The contradiction that arises when you attempt to ascribe causality are laid out (twice) above.

    Don't take up QFT's noncausality with me—take it up with nature. She's the one that's telling us this is the way things work, not me. I'm just the messenger.

    electrons cause photons

    No. The contradiction of this point of view is laid out (twice) above.

    Feser: "even a simple physical phenomenon like the attraction between two particles would suffice for his (Aquinas') purposes. What he is saying is rather that it is impossible that every apparent causal regularity can be attributed to chance, for chance itself presupposes causal regularity."

    This is wrong. QFT is a counterexample. QFT computes the probabilities of chance events without presupposing physical causality.

    You can all call a pragmatic reliance on the real world a "philosophical stance" if you like, but any of the alternatives is a quick path to nonsense or worse. Collectively, the comments here that presuppose you have anything at all to say about the real world without engaging with the facts about how that real world actually works are at once arrogant an ignorant—look at the list of errors compiled in this comment above. At least ignorance is no sin, but willful ignorance is. It's absurd to invent new, obviously incorrect terms like "causality neutral" or ignore the the noncausal nature of quantum field theories.

    No one who knows a little about the real world will take you or your arguments seriously until you take the real world seriously and engage with the facts about the world as we know them, even if that means abandoning personal attachments to ancient and discredited ideas about the world.

    ReplyDelete
  121. Josh:

    >> Does denying the agency of one of the four causes (in this case, the efficient) entail the denial of the agency of all the causes?

    No. If a phenomena is considered to be caused, then it must have all four causes identified. If even a single cause is absent, then the phenomena can be considered uncaused.

    ReplyDelete
  122. >If a phenomena is considered to be caused, then it must have all four causes identified.

    So there really is no such thing as the four causes? It's just one cause?

    >If even a single cause is absent, then the phenomena can be considered uncaused.

    According to whom?

    ReplyDelete
  123. For something to be uncaused that means it happens or actualizes without sufficient conditions and not by any necessity.

    ReplyDelete
  124. Anonymous,

    "Djindra, mangling arguments in the service of wacky political motivation."

    That's the pot calling the kettle black. I draw no political arguments from the bone-headed stupidity of the cosmological argument. Feser, OTOH, does find those political ends, and makes no secret about it. So your accusation, besides being false, is hypocritical.

    ReplyDelete
  125. My previous comment may have gotten missed/ignored during the exchange with Steve Smith.
    I'm wondering if someone more knowledgeable about A-T metaphysics could explain how a B-Theory of time affects the thomistic cosmological argument.
    As I understand it, a B-Theory of time is strongly suggested by relativity, and indicates a 4-dimensional space-time.
    I have an intuition that this may undermine the act-potency distinction being made for the universe which lead to the conclusion of a "first cause" which is outside of this. To me it seems that if the universe doesn't "change" in the intuitive sense (which seems required for the first cause argument), then the argument would fail.

    Is there something I'm missing here, or is this (possibly) a legitimate problem?

    ReplyDelete
  126. Matt Dickinson,

    This blog is not about fairness. And it's not about the denial of preconceptions. We all have them and must have them.

    "But however god is defined in the cosmological argument (excluding personal qualities, benevolence, awareness etc) I do not understand why the same cannot be said of the universe and the words are interchangeable."

    Exactly.

    ReplyDelete
  127. >As I understand it, a B-Theory of time is strongly suggested by relativity,

    Doesn't Eisenstein physics break down at the quantum level? Or something to that effect?

    Personally I believe the whole A vs B theory time dichotomy are both hopelessly limited.

    ReplyDelete
  128. On the matter of the four-dimensionalist universe Feser said "The notion of instrumental causes per se does not become problematic, because even a static four-dimensional block universe would be a compound of essence and existence which would depend for its being on that in which essence and existence are identical.

    Now such a conception of the world -- which is really just a return to Parmenides -- is incompatible with an Aristotelian account of change, precisely because it amounts to the denial of real change. Though I would say that, rather than getting rid of real becoming altogether, it really just relocates it from the world to the mind -- on pain of an incoherent eliminativism -- and thus gives us a Cartesian-style dualistic view of the world, with all the problems of the other kind. But in that case it is like the moderns' attempted elimination of final causes by relocating all "directedness" into the mind -- it doesn't really get rid of the phenomenon the Aristotelian is talking about, but just moves it around.

    Anyway, the four-dimensionalist view is IMO wrong in any case. This is among the subjects of my various current writing projects. More later."END

    ReplyDelete
  129. BenYachov: Doesn't Eisenstein physics break down at the quantum level? Or something to that effect?
    There is currently no quantum theory of gravity, so yes, relativity and QM are not quite compatible at present.
    I don't really see a problem, because QM is time agnostic, as I mentioned, and so it completely compatible with B-Theory.
    I'd really be interested in what the B-Theory would imply for the Thomistic Cosmological Argument (or, at least, the whole act-potency thing, which seems to rely upon change through time).

    BenYachov: Personally I believe the whole A vs B theory time dichotomy are both hopelessly limited.
    How so?

    ReplyDelete
  130. Havok,

    I would invoke as my answer the words of Lee Smolin.

    QUOTE"I have been studying the question of what time is for much of my adult life. But I must admit ... that I am no closer to an answer now than I was then. Indeed, even after all this study, I do not think we can answer even the simple question: 'What sort of thing is time?"

    As such neither A Theory nor B Theory does it for me completely.

    ReplyDelete
  131. Thanks Anonymous.

    Anonymous: On the matter of the four-dimensionalist universe Feser said "The notion of instrumental causes per se does not become problematic, because even a static four-dimensional block universe would be a compound of essence and existence which would depend for its being on that in which essence and existence are identical.
    I don't understand what is meant by that, sorry. I (sort of) understand the act-potency thing, but the essence and existence stuff I'm less comfortable with.

    Anonymous: Though I would say that, rather than getting rid of real becoming altogether, it really just relocates it from the world to the mind -- on pain of an incoherent eliminativism -- and thus gives us a Cartesian-style dualistic view of the world, with all the problems of the other kind.
    I'm not going to argue it here, but I don't buy mind-body dualism of the sort you seem to be talking of, so that doesn't worry me.

    Anonymous: Anyway, the four-dimensionalist view is IMO wrong in any case. This is among the subjects of my various current writing projects. More later.
    As I understand it, it is the view implied strongly by relativity. I also am led to believe it is not friendly to some variations of God belief (William Lane Craig's attempts to formulate his own "replacement" for relativity which favours an A-Theory being an indication).
    Further info would be interesting :-)

    ReplyDelete
  132. Ben:

    >> So there really is no such thing as the four causes? It's just one cause?

    There is such thing as the four causes, but I think that you need all four for something to be considered caused. This is like saying that in order for something to a human, then it must be an animal and have an intellect. If you are missing either of these conditions, then it is not a human. This does not mean that animals and intellects do not exist.

    >> For something to be uncaused that means it happens or actualizes without sufficient conditions and not by any necessity.

    And wouldn’t you say that sufficient conditions require all four causes? Feser writes about the four causes: “In combination, these causes provide a complete explanation of a thing” (Aquinas, p. 16). In other words, a complete explanation requires all the four causes to be accounted for, and without all of them, then the account is necessarily incomplete. The question is whether this incompleteness it due to an event’s being genuinely uncaused, and thus unable to be accounted for within this system, or whether it does have an underlying cause, but we just do not know what it is.

    ReplyDelete
  133. Thanks BenYachov.

    Doesn't that position, coupled with what looks to be at least some questions being raised by a (possible) B-Theory give us (perhaps another) reason to have a little less confidence in a Cosmological Argument which might rely on the particular ontology of time?

    ReplyDelete
  134. Matt:

    >> But however god is defined in the cosmological argument (excluding personal qualities, benevolence, awareness etc) I do not understand why the same cannot be said of the universe and the words are interchangeable.

    That is a good point.

    I think that a Thomist would say that the universe is the sum total of changing events and entities, and thus could not be its own first cause, which is unchanging. However, if you defined the universe as everything that exists, whether necessary or contingent, then you would have to include the first cause as part of the universe. As usual, it depends upon how you define your terms.

    ReplyDelete
  135. FYI

    >I'm not going to argue it here, but I don't buy mind-body dualism of the sort you seem to be talking of, so that doesn't worry me.

    Feser rejects Cartesian-style dualism. Virtually all Thomists do. We hold too HYLEMORPHIC DUALISM.

    http://www.newdualism.org/papers/D.Oderberg/HylemorphicDualism2.htm

    Cheers.

    ReplyDelete
  136. Only if you insist B Theory time must be understood in a way that rejects Moderate Realism and or any real change in things and real change in Time.

    ReplyDelete
  137. BenYachov: We hold too HYLEMORPHIC DUALISM.
    I intended a dualism as being more than a phsyicalist/materialist account of the mind (which seems to include various property dualisms and such), which I think Anonymous was going for.
    I've read that paper by Oderberg before, but can't say I understood it entirely. Perhaps it's time for a reread :-)

    BenYachov: Only if you insist B Theory time must be understood in a way that rejects Moderate Realism and or any real change in things and real change in Time.
    As I understand it, and I could be mistaken, Time becomes something akin to a spatial dimension, which would seem to imply no real change.

    ReplyDelete
  138. >but I think that you need all four for something to be considered caused.

    Is this just your personal opinion? Or did you get it from other Thomist.

    (BTW you are entitled to your opinions just as long as we understand they are not the conclusions of Thomist persay without documentation.)

    More later.

    ReplyDelete
  139. Havok,

    "spatial dimensions" don't ever change?

    How do they even become "spatial dimensions" then in the first place?

    ReplyDelete
  140. Havok anon was directly quoting the words of Edward Feser not giving his own view.

    Just do a google search

    So perhaps you should direct your questions towards him?

    .

    ReplyDelete
  141. BenYachov: "spatial dimensions" don't ever change?
    Without time, I don't think they do/would.

    BenYachov: How do they even become "spatial dimensions" then in the first place?
    Couldn't one say they always were?
    As I understand it, the 4D B-Theory of time is simply eternal and timeless.

    ReplyDelete
  142. BenYachov: Havok anon was directly quoting the words of Edward Feser not giving his own view.
    My mistake. I'd read only the initial paragraph as being Feser.

    BenYachov: So perhaps you should direct your questions towards him?
    I was more after a sense of what implications there might be for the Thomist Cosmological Argument(s) for differing conceptions of time, especially the one which is implied by relativity (and which is not intuitive). I wasn't really too fussed who responded :-)
    I'll track down the post anonymous quoted for a read...

    ReplyDelete
  143. >Without time, I don't think they do/would.

    Doesn't that Beg the question? Does Time really change or not? I don't see why being a spatial dimension inhibits that?

    >Couldn't one say they always were?

    Well a Thomist can believe eternal creation is possible. The Thomistic Cosmological Argument is Top down and would explain an Eternal Universe. That's the point.

    As for Feser answering he is the Thomistic Expert here with the PhD so he is the most qualified.

    Cheers.

    ReplyDelete
  144. BTW

    I really hope nobody here is confusing the Kalam Cosmological Argument with the Thomistic One.

    That is a big no no.

    Mortimer Adler would approve I can tell you that.

    ReplyDelete
  145. BenYachov: Doesn't that Beg the question? Does Time really change or not? I don't see why being a spatial dimension inhibits that?
    As I understand it, it's implies a static, unchanging universe.

    BenYachov: The Thomistic Cosmological Argument is Top down and would explain an Eternal Universe. That's the point.
    From previous comments it seems that an unchanging, "timeless" universe would be problematic.

    BenYachov: As for Feser answering he is the Thomistic Expert here with the PhD so he is the most qualified.
    I'll read his previous post and see if that answers my query.
    Of course, if he happened to chime in, that would be great too :-)

    ReplyDelete
  146. I don't know enough physics to respond & I don't think you know enough Thomism to understand me Havok. So we will leave it at that.

    OTOH taken at face value yours is a way better Atheist critique of the Thomistic Cosmological Argument then the brain dead Gnu shit one reads on Coyne blog.

    I respect that.

    Good job.

    ReplyDelete
  147. BTW dguller you are doing very well too.

    djindra you need help or a hobby or a women.

    Cheers all.

    ReplyDelete
  148. Anonymous: I'm new to this blog. Are bad puns excused after midnight?

    They are excused anytime. After midnight, they're mandatory!

    ReplyDelete
  149. I found this on the Wiki

    QUOTE"It is also common (though not universal) for B-theorists to be four-dimensionalists, that is, to believe that objects are extended in time as well as in space and therefore have temporal as well as spatial parts. This is sometimes called a time-slice ontology (Clark, 1978).

    The debate between A-theorists and B-theorists is a continuation of a metaphysical dispute reaching back to the ancient Greek philosophers Heraclitus and Parmenides. Parmenides thought that reality is timeless and unchanging. Heraclitus, in contrast, believed that the world is a process of ceaseless change, flux and decay. Reality for Heraclitus is dynamic and ephemeral. Indeed the world is so fleeting, according to Heraclitus, that it is impossible to step twice into the same river. The metaphysical issues that continue to divide A-theorists and B-theorists concern the reality of the past, the reality of the future, and the ontological status of the present."END QUOTE

    If this is correct then maybe the whole A Theory vs B Theory concept of time is not compatible with AT in the first place?

    Or maybe Aristotle's solution to Parmenides vs Heraclitus is relevant here?

    This is just my non-expert opinion.

    ReplyDelete
  150. Havok: Would not space-time, when envisaged in that fashion [B-theory], mean that the universe was not in motion and ever changing, that things do not become other things, and therefore Aquinas' premises for his first cause be in difficulty?
If the above is (remotely) correct, wouldn't the universe be a candidate for being "pure act" and therefore Aquinas' first cause?

    Being timeless isn't even close to being pure act, though. Mathematical objects do not change, but the number 6 isn't God. The universe is still contingent — there are limitless possible unchanging universes, so there still will be a cause that explains why we have this particular universe and not that one. (So instead of Aquinas's First Way, we might have to use the Second.) But even if physical objects do not change, our experience of them does; in other words, even if the material universe were static, our consciousness changes (as though it "moved" through the fixed 4-D world), and that incontrovertible change is enough to get the argument going.

    As I understand it, [4-D] is the view implied strongly by relativity. I also am led to believe it is not friendly to some variations of God belief (William Lane Craig's attempts to formulate his own "replacement" for relativity which favours an A-Theory being an indication).


    Well, the physics of relativity works with a 4-D world, but of course that's just the model. All we can say about how the world actually is, is that it must therefore be able to map onto a 4-D framework. Then again, the simplest and most obvious way for the world to do that is for it to really be 4-D; what it boils down to is that reality cannot be "less" than 4-D, but it might be "more" (i.e. there could be aspects of reality that are true, but happen to be irrelevant to doing the phyiscs).

    In particular, relativity tells us that there is no preferred frame of reference — from the phyiscal point of view. That doesn't mean that there cannot be one particular frame which is the "real" one — the "God's-eye perspective" — which defines what we would think of as the "real" line of time in A-theory. It just means that science can't tell us which reference-frame that is. (Obviously, if that frame is special only because of some metaphysical reason like that, there is no reason why physics would or could tell us; the question simply doesn't apply to the scientific method.) Craig's view might be something along this line, I'm not sure; but it's one possible solution.

    ReplyDelete
  151. Dguller: In other words, a complete explanation requires all the four causes to be accounted for, and without all of them, then the account is necessarily incomplete.

    No; a complete explanation requires only all the causes that apply. A material object requires a material cause because it's, well, material! But the number pi does not; it's a pure form, so all it has is a formal cause. You are right, of course, that there may be causes at work in a certain effect that we do not know, or could never discover from the scientific method, or maybe could never find out at all, even in principle. However, if something has at least one cause, then it definitely is correct to say that it is caused.

    ReplyDelete
  152. benYachov: I don't know enough physics to respond & I don't think you know enough Thomism to understand me Havok. So we will leave it at that.
    Agreed. Thanks for your comments.

    ReplyDelete
  153. MrGreen: But even if physical objects do not change, our experience of them does; in other words, even if the material universe were static, our consciousness changes (as though it "moved" through the fixed 4-D world), and that incontrovertible change is enough to get the argument going.
    Doesn't such a position require a specific view of consciousness though?

    ReplyDelete
  154. I note the absence of response to Steve Smith's latest reply. One can only wonder why...

    ReplyDelete
  155. Mr. Smith:

    From your comment of July 22, 2011 2:13 AM (Geez, don't you people have to get up in the morning?):

    ------
    electrons cause photons

    "No. The contradiction of this point of view is laid out (twice) above."
    ------

    I'm curious as to how you think you contradicted this point in your argument of your comment of July 21, 2011 7:23 AM. In your lengthy paragraph describing the appearance of the photon, you wrote:

    Consider the phenomenon of electron-electron repulsion. This is caused by the exchange of a photon between the two electrons, as illustrated in this Feynman diagram (ignore their labels: straight lines are the electrons, wiggly line is the photon). The photon pops just into existence, so what caused it? Well, there are only three particles in this picture, so if the photon was caused by something, it must have been caused one of the two electrons. The Feynman diagram clearly shows that the photon must have been caused by the electron on the left, because we see this electron emit the photon, and then a short time later (up direction in the diagram) the electron on the right absorbs it. So the LHS electron caused the photon. But wait! The rules of QED are invariant to the location of all locations in space-time—in fact, the probability of an event is determined by a sum over all space and time, so we could just as well have drawn the diagram so that it shows the RHS electron emitting the photon and then a short time later the LHS electron absorbing it. So the RHS electron caused the photon.

    Then you draw this conclusion:

    herefore our original assumption that the photon was caused by one of the two electrons is incorrect.

    This conclusion does not follow. There is no case that you presented where neither the LHS nor RHS electron emitted the photon; that is,

    "We could just as well draw the diagram with the LHS electron emitting the photon as we could draw the diagram with the RHS electron emitting the photon"

    !=

    "We could draw the diagram where both electrons emit the photon"

    !=

    "We could draw the diagram with the neither the LHS electron nor the RHS electron emitting the photon, so the photon has no cause"

    In either the first statement (which is what your paragraph said), there is a cause of the photon. The second statement does not leave the photon without a cause either, but with a mind-bending scenario of it being caused simultaneously in two places by two different electrons. How did you get to the third statement from your paragraph?

    ReplyDelete
  156. There is nothing to wonder. It's brain dead Gnu'Atheist anti-philosophical Blather. He's repeating himself.

    Steve could learn a thing or two from dguller or Havok.

    Smeg em!

    ReplyDelete
  157. Mr Green:

    >> However, if something has at least one cause, then it definitely is correct to say that it is caused.

    But we are talking about material entities, and not abstract concepts, such as pi. The dilemma is that there seems to be material entities that are described to an incredibly accurate fashion by a model that does not have to include efficient causes at all for some behaviors of these entities. It is in this context that I am struck by the problematic nature of this for the cosmological argument. If there are such material entities that lack an efficient cause, then I would say that even though they have formal, material and final causes, they are fundamentally uncaused in the relevant sense.

    That being said, as per my typical behavior, I have ordered Feynman’s QED, and have even ordered his Lectures on Physics, as well as some other books on quantum theory to try to get a better understanding. I am open to the possibility that Steve is correct, because I have no vested interest in this fight, being an atheist myself, and not necessarily committed to the cosmological argument. Until they arrive, I am reading some books on physics that have been on my shelf for a while. One of them, “The Universe: A Biography” by John Gribbin has the following passage:

    “When QED was adapted to take account of the presence of this sea of virtual particles, it gave predictions that precisely matched the properties of charged particles measured in experiments … And the agreement is only that good if the effects of quantum uncertainty, a seething vacuum and virtual particles are included” (2008, p. 15).

    This certainly fits my intuitions that the random decay that QED accurately accounts for is rooted in the surrounding field of energy and virtual particles that phase in and out of existence. I find it very hard to believe that it makes more sense that the random decay is more plausibly understood as an ex nihilo uncaused event rather than being the result of the interaction of fermions with vacuum energy and virtual particles, somehow.

    ReplyDelete
  158. djindra, perhaps you are right about this blog, but as a Skeptic please excuse me if I don't take your word for it :)

    ReplyDelete
  159. Dguller:

    "I find it very hard to believe that it makes more sense that the random decay is more plausibly understood as an ex nihilo uncaused event rather than being the result of the interaction of fermions with vacuum energy and virtual particles, somehow."

    I too am interested in the truth on this issue. I started reading QED a few years ago when I was reading all of Feynman's books, but never finished it. Perhaps we can figure this out together, but I think some clarification is needed. Perhaps we could agree on the questions that need to be answered by QED and Feynman?

    1. Is efficient causation provably absent or do we merely have an epistemic limitation at work?

    2. Do the results point to ex nihilo existence?

    And apparently the $64,000 question:

    3. What, if anything, causes the existence of the W boson?

    And philosophically, I'd like your question answered as well by the good Thomists here:

    4. In describing a material body, if one of the causes is absent, does it mean that the body is uncaused?

    I'll be grabbing my copy soon...

    ReplyDelete
  160. Matt, notice how we will engage with people who honestly seek the truth as opposed to those who just wish to score points and repeat the same objections over and over. There was a time when djindra wasn't treated the way he is now.

    ReplyDelete
  161. Josh:

    Those are the key questions that hopefully can be answered by a literature review.

    As for your final question, I would answer that any material phenomena that lacked one of the four causes, especially efficient causation, should be considered uncaused, as per the standard understanding of causality. After all, the efficient causes are what link material entities within space-time from one event to another, and without these interconnecting links, then an event effectively happened without any antecedent event to cause it, which I would consider “uncaused”. You can talk all you want about the other two Aristotelian causes, but that won’t impress me, I’m afraid.

    ReplyDelete
  162. It seems to me that different people are implicitly using different definitions of "cause" here. Let me propose a very loose definition of cause:

    A is a cause for event B if A is a necessary condition for B to occur

    Under this definition, electron-electron repulsion is most definitely caused. It would not occur without the exchange of a photon. The appearance of the photon is also caused. It would not occur without the presence of the pair of electrons.

    It seems to me that the Thomistic cosmological argument holds under this definition of cause.

    ReplyDelete
  163. Havok: Doesn't such a position [experience of change] require a specific view of consciousness though?

    Not really... all it requires is that there is some sort of conscious experience of change. Even if all change is illusory, the "illusion" is changing, in our minds. The only way around that would be to deny that you (or anyone at all) even so much as has a "sense" of change (regardless of what the underlying reality of that change-experience turns out to be). And it's undeniable that we do have such experiences — or at least, if you claimed not to, I don't think anyone would believe you.

    I guess that speaking hypothetically, you could say: let's pretend that there are no human beings, or anything "conscious" that has the experience of different times. Just for the sake of argument, we could suppose a possible world in which the only things that exist are static, eternal, platonic, mathematical objects. (A timeless consciousness would be OK as long as all its experiences — if it has any — too are static, eternal, etc.) In that case, yes, the cosmological argument would be unable to get off the ground. No cosmos, no argument! (Of course, there are other arguments you could use to get to an uncaused cause in such a reality, we just wouldn't call them "cosmological".)

    ReplyDelete
  164. Dear Steve Smith,
    Hah, well, we got you. Seriously, we've all been pulling your leg. You're absolutely right, there is no causality in QFT, but since I think we've put you through enough, I'll let you in on a little secret: there's no "causality" in the Cosmological Argument either. That's just an inside joke. The fact is, the Cosmological Argument is really all about foo. The inevitable foos that permeate the real world logically dictate the existence of an unfoo-ed Foo.

    QFT, of course, is full of foo. If you're not a student of foo, books could be written on the subject, and have been, but in brief, out of the four categories of foo, formal and final foo are particularly relevant here — basically, they are a way to classify patterns: if you have a pattern anywhere, it will be formally and/or finally foo. Does QFT have patterns? Can it calculate, measure, predict the real world? Feynman says yes! So QFT is actually entirely consistent with FT (Foo Theory). Indeed, one of the reasons Aquinas is considered to be one of the great metaphyiscal minds in history, by both theist and atheist alike, is that his theories are fully consistent with modern physics centuries ahead of the fact! (Unlike all those anti-intellectual, anti-science theories that are not consistent with QFT, like, you know, relativity.)

    Anyway, sorry for putting you through all that — just a bit of hazing to which we subject all the newbies. (You'd be amazed how many yutzes we get here who have no interest in understanding the philosophy, only in causing trouble.) But I think you've shown that you're committed to serious rational debate. Sure, our humour may seem a little silly, but don't get worked up over the terminology — at least we didn't name fundamental building blocks of the universe after cheese curds (that come in things like "bottom flavour" — I mean, seriously?!?). No hard feelings!

    ReplyDelete
  165. "I would answer that any material phenomena that lacked one of the four causes, especially efficient causation, should be considered uncaused, as per the standard understanding of causality."

    Ok. Can anyone corroborate this? I'm not exactly confident of my knowledge on this point.

    ReplyDelete
  166. Supposing that it were determined that some subatomic events did come about ex nihilo -- that is, that though the events do fit into some statistical pattern, the characteristics (timing, being reflected or passing through, etc.) of the individual event is, from our standpoint, completely random. Wouldn't it then be the case that it is directly caused by the First Cause? For most of the things we experience, we discern a chain of causes, which we say starts in an uncaused cause. So in these cases, the chain only has one link. After all, the First Cause is supposed to be understood as at all times maintaining all that exists. So maybe this is a glimpse of that maintenance.

    ReplyDelete
  167. Havok

    The following paper should interest you & everyone intrigued by the issues you brought up on B theory time vs
    Aquinas.

    Temporal Parts and the Possibility of
    Change
    DAVID S. ODERBERG

    http://www.reading.ac.uk/AcaDepts/ld/Philos/dso/papers/Temporal%20Parts%20and%20Possibility%20of%20Change.pdf

    hint: Oderberg also rejects four-dimensionalism as incoherent.

    ReplyDelete
  168. Also as Providence would have it the first footnote of Oderberg's paper deals with the issues
    dguller has been grapling with"


    QUOTE"It is sometimes suggested that quantum theory allows for substantial changes out of and
    into nothing, such as the creation and annihilation of pairs of virtual particles, which by
    implication would not involve the survival of anything throughout either event. In reply, it
    must be pointed out (assuming that quantum theory is correct for the purpose of argument)
    that the quantum vacuum out of which and into which such particle pairs emerge
    and vanish is not a literal nothing.
    For instance, Barrow and Tipler say: “It is, of course,
    somewhat inappropriate to call the origin of a bubble Universe in a fluctuation of the
    vacuum ‘creation ex nihilo’, for the quantum mechanical vacuum state has a rich structure
    which resides in a previously existing substratum of space-time, either Minkowski or
    de Sitter space-time.” (The Anthropic Cosmological Principle (Oxford: Clarendon Press,
    1986), p. 441; cited in W.L. Craig and Q. Smith, Theism, Atheism and Big Bang Cosmology
    (Oxford: Clarendon Press, 1993), p. 155.) The cause of virtual particles may be indeterministic
    (licensed by the Uncertainty Principle), but there is still a physical cause consisting
    of the set of physically necessary and sufficient conditions for the particles’
    existence, including the persisting space-time structure itself.
    To say that nothing survives
    the emergence or vanishing of virtual particles is to read more into QM than is actually
    licensed by the theory. The quantum vacuum is not a genuine void. "END QUOTE

    ReplyDelete
  169. If my cut and paste doesn't work simply follow the David Oderberg link here on Feser blog to the paper.

    ReplyDelete
  170. @Mr. Green:

    You Mr. Green are a propagator of heresies. There is no Foo, it is Fu. I know you belong to the Foo school, but the Fu school -- do I really have to list the numerous scholarly articles appearing in magazines as distinguished as the Fu Reviews, The Annals of Fu and other top-notch peer-reviewed journals? -- has shown that there is no Foo, except in the vacant minds of the Foo-ists. You Sir, are clearly lacking in understanding and I suggest you pick up a decent book on Fu -- "The Foo Illusion" by the honorabale Mr. Fuawkins is a great place to start. Please stop this idiotic idea of Foo. It is the essence Fu that permeates the world and keeps it going. You really should stop spreading this Foo nonsense.

    ReplyDelete
  171. Oh shit! Everybody is Kung Foo/Fu fighting!!!!!!

    I'm outta here!!! This will get ugly!!!!!

    ReplyDelete
  172. Ben:

    The problem with that solution is that just because there are physical events surrounding the event in question does not necessarily mean that any of them caused it. Sure, it would be likely that they did, because an ex nihilo event seems ludicrous, but unless you can show that the surrounding physical events caused the event in question, then the matter is an open question at best. At worst, if there are actual models that are incredibly accurate, but that make no mention of those surrounding events at all, then it is questionable whether their presence is the cause of the event in question, and thus it still lacks an efficient cause, which I would argue makes it an uncaused event.

    ReplyDelete
  173. Or it just mean you can't identify the efficient cause.

    ReplyDelete
  174. Meanwhile dguller son you might want to duck.....Kung Foo/Fu fight remember? Not safe!;-)

    ReplyDelete
  175. Ben:

    >> Or it just mean you can't identify the efficient cause.

    You seem to brush this off as not a big deal. The efficient cause is the immediately preceding event that resulted in the effect in question. If there is no such preceding event, then there is no reason why that event happened at the time that it did. And if there is no reason why it happened at that time, then I think it is fair to consider it uncaused.

    You can talk all you want about how its telos was to result in X, and its form was as an entity does X, and it has a material basis. All of which is necessary, but you lack the sufficient condition of an efficient cause, and without it, the explanation is completely empty.

    ReplyDelete
  176. dguller:

    You have thus invoked 'atheism-of-the-gaps', where if scientists can't yet identify what something's efficient cause is, it therefore has no efficient cause, and thus the Cosmological Argument fails.

    Is this really what you want to assert?

    ReplyDelete
  177. >You seem to brush this off as not a big deal.

    It isn't a big deal. I have always believed quantum causality to us is like Primitives on a Pacific Island watching rain come out of a rain cloud.

    They have the primitive concept that somehow that dark cloud "causes" rain to appear even if they don't quite get condensation.

    In a like manner the obvious gap in our knowledge as to why virtual particles arise in the quantum foam isn't a big deal either.

    A "uncaused" of the gaps is no more moving then a God-of-the-gaps which is why I would also hesitate in saying "God did it".

    It's not very Thomist.

    ReplyDelete
  178. Jerry took the words right out of my mouth......

    ReplyDelete
  179. Jerry:

    >> You have thus invoked 'atheism-of-the-gaps', where if scientists can't yet identify what something's efficient cause is, it therefore has no efficient cause, and thus the Cosmological Argument fails.

    Not exactly.

    I am saying that the cosmological argument depends upon the validity of any change from potentially X to actually X requires the intervention of an actual external causal agent. If there are empirical phenomena that make that change from potentially X to actually X in the utter absence of an actual external causal agent, then this key premise is compromised.

    I am not well versed in quantum mechanics and quantum field theory, but it seems that Steve has cited Feynman to the effect that there are quantum phenomena that can fully explained to an incredibly degree of accuracy by a scientific theory that makes absolutely no mention of antecedent causal conditions.

    So what to do here?

    One option is to say that there must be antecedent causal conditions, because those quantum events occur in the context of a surrounding sea of energy fields and virtual particles, which must have some causal impact. However, if they did, then they would be present in the mathematical equations, and the fact that they are not seems to indicate that, although they are present, they are not causal.

    Another option is to say that the fact that this scientific theory is the best confirmed theory in existence, making accurate predictions to a degree beyond other scientific theories, then we should take it very seriously. And the fact that this theory is utterly silent on antecedent conditions means that we have to consider the possibility that there are uncaused natural events that seem to occur quite frequently, in fact.

    It seems to me that the only solution is for someone to demonstrate how the surrounding sea of energy and virtual particles actually cause the subatomic decay in question. And if this is not possible, then I’m afraid that a key premise of the cosmological argument is quite possibly wrong. In other words, it is an impasse, it is an open question, and if it is either of these, then it lacks the foundational support to ground the truth of a metaphysical argument that is supposed to apply to the most general features of reality.

    ReplyDelete
  180. Ben:

    >> It isn't a big deal. I have always believed quantum causality to us is like Primitives on a Pacific Island watching rain come out of a rain cloud.

They have the primitive concept that somehow that dark cloud "causes" rain to appear even if they don't quite get condensation.


    But at least those primitives could point to the dark clouds always preceding the rain, and could realize that rain did not occur in the absence of those clouds. In other words, they may be unable to know the precise mechanisms behind how the clouds cause the rain to fall, but they can see a clear causal relationship

    Compare that to the subatomic decay that Steve described. Other than the very presence of the fermions, is there anything else that is present that distinguishes which fermions will decay and which will not? It does not seem that there is anything at all. I mean, you do not have to give the precise mechanism, but point to something like the rain clouds that always precede the rain. In other words, what antecedent conditions are only present when the decay occurs versus when it does not occur?

    ReplyDelete
  181. dguller Feynman is an even worst philosophical ignoramus than Her Dawkins.

    His motto is "philosophy is bullshit". Which ironically is a philosophical conclusion & not a coherent on at that.

    To many modern physicists are like him. Brilliant physicists kindergarden philosophers if even that.

    Heisenberg(who knew philosophy)he wasn't.

    ReplyDelete
  182. Ben:

    Anything other than an ad hominem attack in your reply?

    ReplyDelete
  183. dguller:
    I wasn't saying that the universe is all that exists (depending what you define as comprising the universe this might not be the case, e.g. the hologram theory), but if the universe is self-actualising, as Feser calls god, then I still think 'god' and the universe are 'interchangeable'. I think I'm pointing out a subtle difference there, but if had already understood that please ignore my implication that you missed it.

    Josh:
    I don't know that djindra isn't engaged with, and I won't take your say so just as I won't take his.
    He was looking for solidarity with me, and I think it fair to have explained to what extent I will give that.
    But I will take your advice with the same friendly intention that I take djindra to have meant his comments, sincere thanks.

    ReplyDelete
  184. >But at least those primitives could point to the dark clouds always preceding the rain, and could realize that rain did not occur in the absence of those clouds.

    I thought you said the "physical cause consisting of the set of physically necessary and sufficient conditions for the particles’ existence, including the persisting space-time structure itself." didn't matter?

    The dark rain cloud is analogous to the space time structure.

    It's obvious.

    Besides non-local hidden variable models by definition are not eliminated. Some QM models assume them.

    ReplyDelete
  185. You know who would be really helpful with this topic? The O Floinn...

    ReplyDelete
  186. >Anything other than an ad hominem attack in your reply?

    Yes Feynman isn't the last word on Quantum Mechanics. Bohm has something to say as well.

    Of course a Quantum Super determinist would poo-poo both of them. Most Christian Physicists I read believe Quantum Super Determinism is a purely Atheistic view.

    ReplyDelete
  187. Ben:

    >> I thought you said the "physical cause consisting of the set of physically necessary and sufficient conditions for the particles’ existence, including the persisting space-time structure itself." didn't matter?

    So, space-time structure is what caused it? Okay, then what accounts for why the space-time structure causes the decay in one fermion, but does not cause it in another fermion?

    If the space-time structure is constant and uniform, then it seems to be present in both conditions, and thus cannot be considered a cause.

    And if the space-time structure is differentiated in some way, then how is it differentiated, and is there an observed relationship between some differentiations in space-time and fermion decay, and other differentiations in space-time and the absence of fermion decay?

    >> Besides non-local hidden variable models by definition are not eliminated. Some QM models assume them.

    Great. What variables cause the decay?

    ReplyDelete
  188. In which case maybe Gerard 't Hooft has the real path to Quantum Atheism and Feynman's Quantum Atheism might be nothing more than YEC for infidels?

    Don't close your mind friends.

    ReplyDelete
  189. dguller,

    We are not going to have a repeat of last time.

    You are not an expert in QM & guess what neither am I. So your scientific questions are wasted one me.

    Well will wait to see if TOF shows up or not.

    That is all.

    ReplyDelete
  190. btw dguller,

    A cloud is not the efficient cause of a raindrop.

    ReplyDelete
  191. OTOH could one be a hyper Calvinist and a Quantum Superdeterminist?

    ReplyDelete
  192. Ben:

    >> You are not an expert in QM & guess what neither am I. So your scientific questions are wasted one me.

    Okay.

    >> A cloud is not the efficient cause of a raindrop.

    But it is the efficient cause of rain, no?

    ReplyDelete
  193. >what accounts for why the space-time structure causes the decay in one fermion, but does not cause it in another fermion?

    Primitive 1: Why do some clouds produce rain & some snow? Why doesn't the whole cloud turn into liquid water if drops truly come from the cloud?

    Primitive 2: It's the choice of the gods.

    Primitive 3: There are no gods rain drops appear uncaused in the cloud.

    Primitive djindra: I don't know but it serves Primitive Feser's political agenda!

    ;-)

    ReplyDelete
  194. >But it is the efficient cause of rain, no?

    If so then a Quantum vacuum is the efficient cause of Quantum Events.

    ReplyDelete
  195. Anyway dguller remember your bias.

    I'll remember mine & we shall brush up on Quantum physics and philosophy.

    ReplyDelete
  196. Primitive dguller: I must examine my bias and learn more.

    Primitive BenYachov: Will one of you science wannbes invent an Xbox 360 already! This Island is boring and I want to play Fallout New Vegas!

    F***! I hate looking at clouds!

    :D

    ReplyDelete
  197. dguller:

    ...it seems that Steve has cited Feynman to the effect that there are quantum phenomena that can fully explained to an incredibly degree of accuracy by a scientific theory that makes absolutely no mention of antecedent causal conditions.

    What he wrote was that QFT

    "computes the probabilities of chance events without presupposing physical causality."

    So because it leaves it out, it therefore does not exist?

    Could you not compute the probabilities of a series of coin-flips or dice-rolls without presupposing physical causality..and without leaping to the conclusion that the coin-flips or dice-rolls were uncaused?

    As to your question about fermion decay (July 22, 2011 1:58 PM), -- overlooking that for scenario to prove what you want it to, you would have to stipulate that both fermions are identical and occupy the same point in space -- you seem to be confusing "indeterminate" and "uncaused". I think in your planned Quantum reading that you'll come to appreciate the difference.

    Finally, as much as you will hate to hear this, BenYachov has a point: Fenyman is not the last word on Quantum Physics. I believe Appendix A and B of "Modern Physics and Ancient Faith" by Dr. Stephen Barr (of the Bartol Research Institute For Physics at the U. of Delaware) addresses questions about Aquinas, Quantum Physics, and "Creation from nothing".

    As much as I'd like to bat this around all day....well, I wouldn't want to bat this around all day. Where on earth do you guys get the time?

    JB

    ReplyDelete
  198. Jake: It seems to me that different people are implicitly using different definitions of "cause" here. Let me propose a very loose definition of cause:
    A is a cause for event B if A is a necessary condition for B to occur


    Almost, although in Thomistic terminology, "occur" is a bit too narrow, since causes apply even to things outside time. There are reasons — necessary conditions that must obtain — for Pythagoras's Theorem to be true, so even though abstract platonic triangles do not change or exist in time, there are still causes. But "occurrence" does apply to the physical realm, and your conclusion about the causation of the photon is right.

    As to whether all four causes must apply to physical objects... well, by definition a material object consists of matter joined to form, so it must have at least those two causes. Could you have matter with no final cause? I'm suspicious of that, but maybe you could in theory — the thing is, final causality is how a thing behaves, so a finally-causeless object would be one that didn't do anything. In which case you couldn't even tell it was there; it would be completely invisible to any physical observation, because if it reacted to light, or charge, or anything else, then that's a kind of behaviour and so it would have a final cause. So not much point worrying about things that are irrelevant, especially to physics itself.

    As for whether every physical object requires an efficient cause, in some sense it will have to have one. Normally we think of them as being brought about by the action of some other physical objects, according to their essences and tendencies (formal+final causes). But even if there were no such secondary physical efficient cause, there would still be the direct act of God in uniting the resulting object's essence and existence. In fact, as Aquinas deduces elsewhere, God must always be acting in such a way as a primary cause (i.e. maintaing everything in existence, which is the point SR was making above). So there would be always be an efficient cause, just not one that necessarily has any relevance to physics qua science.

    ReplyDelete
  199. Dguller: The problem with that solution is that just because there are physical events surrounding the event in question does not necessarily mean that any of them caused it. [...]
    At worst, if there are actual models that are incredibly accurate, but that make no mention of those surrounding events at all, then it is questionable whether their presence is the cause of the event in question, and thus it still lacks an efficient cause, which I would argue makes it an uncaused event.


    Indeed, God could (miraculously) cause something to pop up in the middle of a physical event that was uncaused by any of the surrounding objects, but of course then God is the (primary) cause and there simply are no secondary causes. Now as you say, if the physics made no mention of this (as it wouldn't in the case of a miracle out of nowhere) then we could conclude that it was physically uncaused, i.e. no secondary physical causes at work. [Technically, that also assumes that the science of physics is capable of detecting all physical events, which doesn't have to be true. There can also be things which science doesn't detect but could if we had better equipment/experiments, etc.]

    The efficient cause is the immediately preceding event that resulted in the effect in question. If there is no such preceding event, then there is no reason why that event happened at the time that it did. And if there is no reason why it happened at that time, then I think it is fair to consider it uncaused.

    Physically uncaused, and in the examples under consideration, physically efficiently uncaused. The formal and final causes are still at work, or we couldn't do any physics with the event (and we can, e.g. even though we can't predict the exact time that a particular atom of uranium will decay, we can predict the time it takes for half the atoms to decay, and we can predict that if there were no uranium there, there wouldn't be any decay! So there is at least some physical cause, and at least the primary cause (essence+existence) that there always is. Hence, not uncaused.

    It seems to me that the only solution is for someone to demonstrate how the surrounding sea of energy and virtual particles actually cause the subatomic decay in question.

    What do you mean by "how"? Once we have described all the causes that apply, primary and secondary, physical and non-physical, material/formal/final/efficient, then that's all the explanation there is, or can be.

    ReplyDelete